Решение сложных пределов: Сложные пределы и методы их решения

Содержание

Методы решения пределов в картинках

Приводятся методы, применяемые при вычислении пределов функций в сжатом виде – в виде изображений. Каждая картинка содержит основные формулы и понятия страницы, к которой она относится. Картинки сопровождаются заголовками, описаниями страниц и ссылками на них.

Здесь приводится содержание раздела «Методы вычисления пределов» в картинках. На изображениях, в кратком виде представлено содержание страниц раздела. На многих из них излагаются методы, применяемые при вычислении пределов. Рядом с каждым изображением имеется заголовок, описание страницы и ссылка на нее. Просматривая их, можно освежить в памяти применяемые методы и некоторые формулы, а также перейти на страницу с подробным изложением материала.

Методы вычисления пределов функций и раскрытия неопределенностей

Изложены приемы и методы решения задач на вычисление пределов и раскрытие неопределенностей. Рассмотрены следующие вопросы: пределы с непрерывными и сложными функциями; известные пределы; сведение неопределенности одного вида к другому; раскрытие неопределенностей с дробями из многочленов и корней; сравнение функций и решение разложением в степенной ряд; правило Лопиталя.

Примеры пределов с решениями

Страница содержит ссылки на 45 примеров решений пределов функций и 24 задачи на смежные темы. К смежным темам относятся задачи на применение определений предела последовательности и предела функции, а также задачи на непрерывность функции.

Замена переменной при решении пределов

Изложены правила, которые необходимо соблюдать, применяя замену переменной при решении пределов. Формальное применение подстановок, в некоторых случаях, может приводить к неверному результату. Приводится пример, в котором существуют промежуточные пределы, но предела исходной сложной функции не существует.

Решение пределов с дробями из многочленов

Изложены приемы и методы решения пределов дробей с отношениями многочленов. Рассмотрены неопределенности вида ∞ / ∞, 0 / 0 и ∞ ± ∞. Разобраны случаи, когда переменная стремится к бесконечности и к конечному числу. Для каждого варианта приводятся примеры с подробными объяснениями и ссылками на применяемые теоремы и свойства.

Решение пределов с корнями

Изложены методы решения задач на вычисление пределов и раскрытие неопределенностей от функций с корнями. Рассмотрены следующие приемы: применение подстановки; применение формул разности квадратов (и других степеней) для линеаризации бесконечно малой части; деление числителя и знаменателя дроби на степенную функцию. Приводятся примеры с подробными решениями.

Доказательство первого замечательного предела и его следствий

Приводится доказательство первого замечательного предела и его следствий. Дается определение длины дуги окружности как верхней грани множества длин ломаных, вписанных в дугу.

Примеры решений задач с помощью первого замечательного предела

Собраны формулы, свойства и теоремы, применяемые при решении задач, допускающих решение с помощью первого замечательного предела. Даны подробные решения примеров с использованием первого замечательного предела и его следствий.

Доказательство второго замечательного предела и его следствий

Приводится доказательство второго замечательного предела и его следствий.

Примеры решений задач с помощью второго замечательного предела

Подробные решения примеров с использованием второго замечательного предела и его следствий. Формулы, свойства и теоремы, применяемые при решении задач, допускающих решение с помощью второго замечательного предела.

О большое и о малое. Сравнение функций

Даны определения о малого, о большого, эквивалентных (асимптотически равных) функций, функций одного порядка, и их свойства. Приводятся доказательства свойств и теорем. Эти свойства и теоремы используются для сравнения функций и вычисления пределов при аргументе, стремящемся к конечной или бесконечно удаленной точке.

Решение пределов, используя ряд Тейлора

Изложен метод решения пределов, используя разложение функций в ряд Тейлора. Приводятся применяемые в этом методе свойства о малого и разложения элементарных функций в ряд Маклорена. Подробно разобраны примеры решения пределов, содержащих неопределенности ∞ – ∞, один в степени бесконечность и 0/0.

Решение пределов функций, используя правило Лопиталя

Изложен метод решения пределов, используя правило Лопиталя. Приводятся формулировки соответствующих теорем. Подробно разобраны примеры решения пределов, содержащих неопределенности ∞/∞, 0/0, 0 в степени 0 и ∞ – ∞, с помощью правила Лопиталя.

Применение эквивалентных функций при решении пределов

Изложен метод, позволяющий упростить вычисление пределов, применяя эквивалентные функции. Этот метод применим при вычислении пределов дробей с множителями в числителе или знаменателе. Дана таблица эквивалентных функций при x→0. Приводятся подробно разобранные примеры применения этого метода.

Правило Лопиталя: теория и примеры решений

  • Правило Лопиталя и раскрытие неопределённостей
  • Раскрытие неопределённостей видов «ноль делить на ноль» и «бесконечность делить на бесконечность»
  • Применить правило Лопиталя самостоятельно, а затем посмотреть решение
  • Раскрытие неопределённостей вида «ноль умножить на бесконечность»
  • Раскрытие неопределённостей видов «ноль в степени ноль», «бесконечность в степени ноль» и «один в степени бесконечность»
  • Раскрытие неопределённостей вида «бесконечность минус бесконечность»

Производная от функции недалеко падает, а в случае правил Лопиталя она падает точно туда же, куда падает исходная функция. Это обстоятельство помогает в раскрытии неопределённостей вида 0/0 или ∞/∞ и некоторых других неопределённостей, возникающих при вычислении предела отношения двух бесконечно малых или бесконечно больших функций. Вычисление значительно упрощается с помощью этого правила (на самом деле двух правил и замечаний к ним):

.

Как показывает формула выше, при вычислении предела отношений двух бесконечно малых или бесконечно больших функций предел отношения двух функций можно заменить пределом отношения их производных и, таким образом, получить определённный результат.

Перейдём к более точным формулировкам правил Лопиталя.

Правило Лопиталя для случая предела двух бесконечно малых величин. Пусть функции f(x) и g(x) имеют производные (то есть дифференцируемы) в некоторой окрестности точки a. А в самой точке a они могут и не иметь производных. При этом в окрестности точки

a производная функции g(x) не равна нулю (g‘(x)≠0) и пределы этих функций при стремлении икса к значению функции в точке a равны между собой и равны нулю:

.

Тогда предел отношения этих функций равен пределу отношения их производных:

.

Правило Лопиталя для случая предела двух бесконечно больших величин. Пусть функции f(x) и g(x) имеют производные (то есть дифференцируемы) в некоторой окрестности точки a. А в самой точке a они могут и не иметь производных. При этом в окрестности точки a производная функции g(x) не равна нулю (g‘(x)≠0) и пределы этих функций при стремлении икса к значению функции в точке a равны между собой и равны бесконечности:

.

Тогда предел отношения этих функций равен пределу отношения их производных:

.

Иными словами, для неопределённостей вида 0/0 или ∞/∞ предел отношения двух функций равен пределу отношения их производных, если последний существует (конечный, то есть равный определённому числу, или бесконечный, то есть равный бесконечности).

Замечания.

1. Правила Лопиталя применимы и тогда, когда функции f(x) и g(x) не определены при x = a.

2. Если при вычисления предела отношения производных функций f(x) и g(x) снова приходим к неопределённости вида 0/0 или ∞/∞, то правила Лопиталя следует применять многократно (минимум дважды).

3. Правила Лопиталя применимы и тогда, когда аргумент функций (икс) стремится не к конечному числу

a, а к бесконечности (x → ∞).

К неопределённостям видов 0/0 и ∞/∞ могут быть сведены и неопределённости других видов.


  • Пригодится: тригонометрические тождества для преобразования выражений

Пример 1. Вычислить предел отношения двух функций, пользуясь правилом Лопиталя:

Решение. Подстановка в заданную функцию значения x=2 приводит к неопределённости вида 0/0. Поэтому производную каждой функции и получаем

В числителе вычисляли производную многочлена (применяя для этого формулы 1, 2 и 3 из таблицы производных), а в знаменателе — производную сложной логарифмической функции. Перед последним знаком равенства вычисляли обычный

предел, подставляя вместо икса двойку.

Пример 2. Вычислить предел отношения двух функций, пользуясь правилом Лопиталя:

.

Решение. Подстановка в заданную функцию значения x=0 приводит к неопределённости вида 0/0. Поэтому вычисляем производные функций в числителе и знаменателе и получаем:

Пример 3. Вычислить предел отношения двух функций, пользуясь правилом Лопиталя:

.

Решение. Подстановка в заданную функцию значения x=0 приводит к неопределённости вида 0/0. Поэтому вычисляем производные функций в числителе и знаменателе и получаем:

Пример 4. Вычислить

.

Решение. Подстановка в заданную функцию значения икса, равного плюс бесконечности, приводит к неопределённости вида ∞/∞. Поэтому применим правило Лопиталя:


Замечание. Переходим к примерам, в которых правило Лопиталя приходится применять дважды, то есть приходить к пределу отношений вторых производных, так как предел отношения первых производных представляет собой неопределённость вида 0/0 или ∞/∞.

Пример 5. Вычислить предел отношения двух функций, пользуясь правилом Лопиталя:

.

Решение. Находим

Здесь правило Лопиталя применено дважды, поскольку и предел отношения функций, и предел отношения производных дают неопределённость вида ∞/∞.

Пример 6. Вычислить

.

Решение. Находим

Здесь правило Лопиталя применено дважды, поскольку и предел отношения функций, и предел отношения производных дают неопределённость вида 0/0.

Нет времени вникать в решение? Можно заказать работу!

Пример 7. Вычислить

.

Решение. Находим

Здесь правило Лопиталя применено дважды, поскольку и предел отношения функций, и предел отношения производных сначала дают неопределённость вида — ∞/∞, а затем неопределённость вида 0/0.

Пример 8. Вычислить

.

Решение. Находим

Здесь правило Лопиталя применено дважды, поскольку и предел отношения функций, и предел отношения производных сначала дают неопределённость вида ∞/∞, а затем неопределённость вида 0/0.

  • Пригодится: тригонометрические тождества для преобразования выражений

Пример 9. Вычислить

.

Подсказка. Здесь придётся попыхтеть несколько больше обычного над преобразованием выражений под знаком предела.

Посмотреть правильное решение и ответ.

Пример 10. Вычислить

.

Подсказка. Здесь правило Лопиталя придётся применять трижды.

Посмотреть правильное решение и ответ.


  • Пригодится: тригонометрические тождества для преобразования выражений

Пример 11. Вычислить

.

Решение. Получаем

(здесь неопределённость вида 0∙∞ мы преобразовали к виду ∞/∞, так как

а затем применили правила Лопиталя).

Пример 12. Вычислить

.

Решение. Получаем

В этом примере использовано тригонометрическое тождество .

  • Пригодится: тригонометрические тождества для преобразования выражений

Неопределённости вида , или обычно приводятся к виду 0/0 или ∞/∞ с помощью логарифмирования функции вида

Чтобы вычислить предел выражения , следует использовать логарифмическое тождество , частным случаем которого является и свойство логарифма .

Используя логарифмическое тождество и свойство непрерывности функции (для перехода за знак предела), предел следует вычислять следующим образом:

Отдельно следует находить предел выражения в показателе степени и возводить e в найденную степень.

Пример 13. Вычислить, пользуясь правилом Лопиталя

.

Решение. Получаем

Вычисляем предел выражения в показателе степени

.

Итак,

.

Пример 14. Вычислить, пользуясь правилом Лопиталя

.

Решение. Получаем

Вычисляем предел выражения в показателе степени

.

Итак,

.

Пример 15. Вычислить, пользуясь правилом Лопиталя

.

Решение. Получаем

Вычисляем предел выражения в показателе степени

Итак,

.

  • Пригодится: тригонометрические тождества для преобразования выражений

Это случаи, когда вычисление предела разности функций приводит к неопределённости «бесконечность минус бесконечность»: .

Вычисление такого предела по правилу Лопиталя в общем виде выглядит следующим образом:

В результате таких преобразований часто получаются сложные выражения, поэтому целесообразно использовать такие преобразования разности функций, как приведение к общему знаменателю, умножение и деление на одно и то же число, использование тригонометрических тождеств и т.д.

Пример 16. Вычислить, пользуясь правилом Лопиталя

.

Решение. Пользуясь вышеперечисленными рекомендациями, получаем

Пример 17. Вычислить, пользуясь правилом Лопиталя

.

Решение. Пользуясь вышеперечисленными рекомендациями, получаем

  • Пригодится: тригонометрические тождества для преобразования выражений
НазадЛистатьВперёд>>>

Нет времени вникать в решение? Можно заказать работу!

К началу страницы

Пройти тест по теме Производная, дифференциал и их применение

Весь блок «Производная»

  • Что такое производная
  • Найти производную: алгоритм и примеры решений
  • Производные произведения и частного функций
  • Производная суммы дробей со степенями и корнями
  • Производные простых тригонометрических функций
  • Производная сложной функции
  • Производная логарифмической функции
  • Дифференциал функции
  • Дифференциал сложной функции, инвариантность формы дифференциала
  • Уравнение касательной и уравнение нормали к графику функции
  • Правило Лопиталя
  • Частные производные

Методы решения пределов.

Неопределённости.Порядок роста функции

Предел функции — число a будет пределом некоторой изменяемой величины, если в процессе своего изменения эта переменная величина неограниченно приближается к a .

Или другими словами, число A является пределом функции y = f (x) в точке x 0 , если для всякой последовательности точек из области определения функции , не равных x 0 , и которая сходится к точке x 0 (lim x n = x0) , последовательность соответствующих значений функции сходится к числу A .

График функции, предел которой при аргументе, который стремится к бесконечности, равен L :

Значение А является пределом (предельным значением) функции f (x) в точке x 0 в случае, если для всякой последовательности точек , которая сходится к x 0 , но которая не содержит x 0 как один из своих элементов (т.е. в проколотой окрестности x 0 ), последовательность значений функции сходится к A .

Предел функции по Коши.

Значение A будет являться пределом функции f (x) в точке x 0 в случае, если для всякого вперёд взятого неотрицательного числа ε будет найдено соответствующее ему неотрицательно число δ = δ(ε) такое, что для каждого аргумента x , удовлетворяющего условию 0 , будет выполнено неравенство | f (x) A | .

Будет очень просто, если вы понимаете суть предела и основные правила нахождения его. То, что предел функции f (x) при x стремящемся к a равен A , записывается таким образом:

Причем значение, к которому стремится переменная x , может быть не только числом, но и бесконечностью (∞), иногда +∞ или -∞, либо предела может вообще не быть.

Чтоб понять, как находить пределы функции , лучше всего посмотреть примеры решения.

Необходимо найти пределы функции f (x) = 1/ x при:

x → 2, x → 0, x ∞.

Найдем решение первого предела. Для этого можно просто подставить вместо x число, к которому оно стремится, т.е. 2, получим:

Найдем второй предел функции . Здесь подставлять в чистом виде 0 вместо x нельзя, т.к. делить на 0 нельзя. Но мы можем брать значения, приближенные к нулю, к примеру, 0,01; 0,001; 0,0001; 0,00001 и так далее, причем значение функции f (x) будет увеличиваться: 100; 1000; 10000; 100000 и так далее. Т.о., можно понять, что при x → 0 значение функции, которая стоит под знаком предела, будет неограниченно возрастать, т.е. стремиться к бесконечности. А значит:

Касаемо третьего предела. Такая же ситуация, как и в прошлом случае, невозможно подставить в чистом виде. Нужно рассмотреть случай неограниченного возрастания x . Поочередно подставляем 1000; 10000; 100000 и так далее, имеем, что значение функции f (x) = 1/ x будет убывать: 0,001; 0,0001; 0,00001; и так далее, стремясь к нулю. Поэтому:

Необходимо вычислить предел функции

Приступая к решению второго примера, видим неопределенность . Отсюда находим старшую степень числителя и знаменателя — это x 3 , выносим в числителе и знаменателе его за скобки и далее сокращаем на него:

Ответ

Первым шагом в нахождении этого предела , подставим значение 1 вместо x , в результате чего имеем неопределенность . Для её решения разложим числитель на множители , сделаем это методом нахождения корней квадратного уравнения x 2 + 2 x — 3 :

D = 2 2 — 4*1*(-3) = 4 +12 = 16 D = √16 = 4

x 1,2 = (-2 ± 4) / 2 x 1 = -3; x 2 = 1.

Таким образом, числитель будет таким:

Ответ

Это определение его конкретного значения или определенной области, куда попадает функция, которая ограничена пределом.

Чтобы решить пределы, следуйте правилам:

Разобравшись в сути и основных правилах решения предела , вы получите базовое понятие о том, как их решать.

Определение пределов последовательности и функции, свойства пределов, первый и второй замечательные пределы, примеры.

Постоянное число а называется пределом последовательности {x n }, если для любого сколь угодно малого положительного числа ε > 0 существует номер N, что все значения x n , у которых n>N, удовлетворяют неравенству

Записывают это следующим образом: или x n → a.

Неравенство (6.1) равносильно двойному неравенству

a — ε x n , начиная с некоторого номера n>N, лежат внутри интервала (a-ε , a+ε), т.е. попадают в какую угодно малую ε-окрестность точки а .

Последовательность, имеющая предел, называется сходящейся , в противном случае — расходящейся .

Понятие предел функции является обобщением понятия предел последовательности, так как предел последовательности можно рассматривать как предел функции x n = f(n) целочисленного аргумента n .

Пусть дана функция f(x) и пусть a предельная точка области определения этой функции D(f), т. е. такая точка, любая окрестность которой содержит точки множества D(f), отличные от a . Точка a может принадлежать множеству D(f), а может и не принадлежать ему.

Определение 1. Постоянное число А называется предел функции f(x) при x→ a, если для всякой последовательности {x n } значений аргумента, стремящейся к а , соответствующие им последовательности {f(x n)} имеют один и тот же предел А.

Это определение называют определением предела функции по Гейне, или “на языке последовательностей ”.

Определение 2 . Постоянное число А называется предел функции f(x) при x→a, если, задав произвольное, как угодно малое положительное число ε, можно найти такое δ >0 (зависящее от ε), что для всех x , лежащих в ε-окрестности числа а , т.е. для x , удовлетворяющих неравенству
0

Это определение называют определением предел функции по Коши, или “на языке ε — δ «

Определения 1 и 2 равносильны. Если функция f(x) при x → a имеет предел , равный А, это записывается в виде

В том случае, если последовательность {f(x n)} неограниченно возрастает (или убывает) при любом способе приближения x к своему пределу а , то будем говорить, что функция f(x) имеет бесконечный предел, и записывать это в виде:

Переменная величина (т.е. последовательность или функция), предел которой равен нулю, называется бесконечно малой величиной.

Переменная величина, предел которой равен бесконечности, называется бесконечно большой величиной .

Чтобы найти предел на практике пользуются следующими теоремами.

Теорема 1 . Если существует каждый предел

(6.4)

(6.5)

(6.6)

Замечание . Выражения вида 0/0, ∞/∞, ∞-∞ 0*∞ являются неопределенными, например, отношение двух бесконечно малых или бесконечно больших величин, и найти предел такого вида носит название “раскрытие неопределенностей”.

Теорема 2.

т.е. можно переходить к пределу в основании степени при постоянном показателе, в частности,

Теорема 3.

(6.11)

где e » 2.7 — основание натурального логарифма. Формулы (6.10) и (6.11) носят название первый замечательного предело и второй замечательный предел.

Используются на практике и следствия формулы (6.11):

(6.12)

(6.13)

(6.14)

в частности предел,

Eсли x → a и при этом x > a, то пишут x →a + 0. Если, в частности, a = 0, то вместо символа 0+0 пишут +0. Аналогично если x→a и при этом x и называются соответственно предел справа и предел слева функции f(x) в точке а . Чтобы существовал предел функции f(x) при x→ a необходимо и достаточно, чтобы . Функция f(x) называется непрерывной в точке x 0 , если предел

(6.15)

Условие (6.15) можно переписать в виде:

то есть возможен предельный переход под знаком функции, если она непрерывна в данной точке.

Если равенство (6.15) нарушено, то говорят, что при x = x o функция f(x) имеет разрыв. Рассмотрим функцию y = 1/x. Областью определения этой функции является множество R , кроме x = 0. Точка x = 0 является предельной точкой множества D(f), поскольку в любой ее окрестности, т.е. в любом открытом интервале, содержащем точку 0, есть точки из D(f), но она сама не принадлежит этому множеству. Значение f(x o)= f(0) не определено, поэтому в точке x o = 0 функция имеет разрыв.

Функция f(x) называется непрерывной справа в точке x o , если предел

и непрерывной слева в точке x o, если предел

Непрерывность функции в точке x o равносильна ее непрерывности в этой точке одновременно и справа и слева.

Для того, чтобы функция была непрерывна в точке x o , например, справа, необходимо, во-первых, чтобы существовал конечный предел , а во-вторых, чтобы этот предел был равен f(x o). Следовательно, если хотя бы одно из этих двух условий не выполняется, то функция будет иметь разрыв.

1. Если предел существует и не равен f(x o), то говорят, что функция f(x) в точке x o имеет разрыв первого рода, или скачок .

2. Если предел равен +∞ или -∞ или не существует, то говорят, что в точке x o функция имеет разрыв второго рода .

Например, функция y = ctg x при x → +0 имеет предел, равный +∞ , значит, в точке x=0 она имеет разрыв второго рода. Функция y = E(x) (целая часть от x ) в точках с целыми абсциссами имеет разрывы первого рода, или скачки.

Функция, непрерывная в каждой точке промежутка , называется непрерывной в . Непрерывная функция изображается сплошной кривой.

Ко второму замечательному пределу приводят многие задачи, связанные с непрерывным ростом какой-либо величины. К таким задачам, например, относятся: рост вклада по закону сложных процентов, рост населения страны, распад радиоактивного вещества, размножение бактерий и т.п.

Рассмотрим пример Я. И. Перельмана , дающий интерпретацию числа e в задаче о сложных процентах. Число e есть предел . В сбербанках процентные деньги присоединяются к основному капиталу ежегодно. Если присоединение совершается чаще, то капитал растет быстрее, так как в образовании процентов участвует большая сумма. Возьмем чисто теоретический, весьма упрощенный пример. Пусть в банк положено 100 ден. ед. из расчета 100 % годовых. Если процентные деньги будут присоединены к основному капиталу лишь по истечении года, то к этому сроку 100 ден. ед. превратятся в 200 ден.ед. Посмотрим теперь, во что превратятся 100 ден. ед., если процентные деньги присоединять к основному капиталу каждые полгода. По истечении полугодия 100 ден. ед. вырастут в 100 ×1,5 = 150, а еще через полгода — в 150× 1,5 = 225 (ден. ед.). Если присоединение делать каждые 1/3 года, то по истечении года 100 ден. ед. превратятся в 100 × (1 +1/3) 3 ≈ 237 (ден. ед.). Будем учащать сроки присоединения процентных денег до 0,1 года, до 0,01 года, до 0,001 года и т.д. Тогда из 100 ден. ед. спустя год получится:

100×(1 +1/10) 10 ≈ 259 (ден. ед.),

100×(1+1/100) 100 ≈ 270 (ден. ед.),

100×(1+1/1000) 1000 ≈271 (ден. ед.).

При безграничном сокращении сроков присоединения процентов наращенный капитал не растет беспредельно, а приближается к некоторому пределу, равному приблизительно 271. Более чем в 2,71 раз капитал, положенный под 100% годовых, увеличиться не может, даже если бы наросшие проценты присоединялись к капиталу каждую секунду, потому что предел

Пример 3.1 . Пользуясь определением предела числовой последовательности, доказать, что последовательность x n =(n-1)/n имеет предел, равный 1.

Решение. Нам надо доказать, что, какое бы ε > 0 мы ни взяли, для него найдется натуральное число N, такое, что для всех n > N имеет место неравенство |x n -1|

Возьмем любое ε > 0. Так как x n -1 =(n+1)/n — 1= 1/n, то для отыскания N достаточно решить неравенство 1/n1/ε и, следовательно, за N можно принять целую часть от 1/ε N = E(1/ε). Мы тем самым доказали, что предел .

Пример 3.2. Найти предел последовательности, заданной общим членом .

Решение. Применим теорему предел суммы и найдем предел каждого слагаемого. При n → ∞ числитель и знаменатель каждого слагаемого стремится к бесконечности, и мы не можем непосредственно применить теорему предел частного. Поэтому сначала преобразуем x n , разделив числитель и знаменатель первого слагаемого на n 2 , а второго на n . Затем, применяя теорему предел частного и предел суммы, найдем:

Пример 3.3 . . Найти .

Решение.

Здесь мы воспользовались теоремой о пределе степени: предел степени равен степени от предела основания.

Пример 3.4 . Найти ().

Решение. Применять теорему предел разности нельзя, поскольку имеем неопределенность вида ∞-∞. Преобразуем формулу общего члена:

Пример 3.5 . Дана функция f(x)=2 1/x . Доказать, что предел не существует.

Решение. Воспользуемся определением 1 предела функции через последовательность. Возьмем последовательность { x n }, сходящуюся к 0, т.е. Покажем, что величина f(x n)= для разных последовательностей ведет себя по-разному. Пусть x n = 1/n. Очевидно, что , тогда предел Выберем теперь в качестве x n последовательность с общим членом x n = -1/n, также стремящуюся к нулю. Поэтому предел не существует.

Пример 3.6 . Доказать, что предел не существует.

Решение. Пусть x 1 , x 2 ,…, x n ,… — последовательность, для которой
. Как ведет себя последовательность {f(x n)} = {sin x n } при различных x n → ∞

Если x n = p n, то sin x n = sin (p n) = 0 при всех n и предел Если же
x n =2 p n+ p /2, то sin x n = sin(2 p n+ p /2) = sin p /2 = 1 для всех n и следовательно предел . Таким образом, не существует.

Пример 4

Найти предел

Это более простой пример для самостоятельного решения. В предложенном примере снова неопределённость ( более высокого порядка роста, чем корень ).

Если «икс» стремится к «минус бесконечности»

Призрак «минус бесконечности» уже давно витал в этой статье. Рассмотрим пределы с многочленами, в которых . Принципы и методы решения будут точно такими же, что и в первой части урока, за исключением ряда нюансов.

Рассмотрим 4 фишки, которые потребуются для решения практических заданий:

1) Вычислим предел

Значение предела зависит только от слагаемого , поскольку оно обладает самым высоким порядком роста. Если , то бесконечно большое по модулю отрицательное число в ЧЁТНОЙ степени , в данном случае – в четвёртой, равно «плюс бесконечности»: . Константа («двойка») положительна , поэтому:

2) Вычислим предел

Здесь старшая степень опять чётная , поэтому: . Но перед расположился «минус» (отрицательная константа –1), следовательно:

3) Вычислим предел

Значение предела зависит только от . Как вы помните из школы, «минус» «выскакивает» из-под нечётной степени, поэтому бесконечно большое по модулю отрицательное число в НЕЧЁТНОЙ степени равно «минус бесконечности», в данном случае: .
Константа («четвёрка») положительна , значит:

4) Вычислим предел

Первый парень на деревне снова обладает нечётной степенью, кроме того, за пазухой отрицательная константа, а значит: Таким образом:
.

Пример 5

Найти предел

Используя вышеизложенные пункты, приходим к выводу, что здесь неопределённость . Числитель и знаменатель одного порядка роста, значит, в пределе получится конечное число. Узнаем ответ, отбросив всех мальков:

Решение тривиально:

Пример 6

Найти предел

Это пример для самостоятельного решения. Полное решение и ответ в конце урока.

А сейчас, пожалуй, самый тонкий из случаев:

Пример 7

Найти предел

Рассматривая старшие слагаемые, приходим к выводу, что здесь неопределённость . Числитель более высокого порядка роста, чем знаменатель, поэтому сразу можно сказать, что предел равен бесконечности. Но какой бесконечности, «плюс» или «минус»? Приём тот же – в числителе и знаменателе избавимся от мелочи:

Решаем:

Разделим числитель и знаменатель на

Пример 15

Найти предел

Это пример для самостоятельного решения. Примерный образец чистового оформления в конце урока.

Ещё пара занятных примеров на тему замены переменной:

Пример 16

Найти предел

При подстановке единицы в предел получается неопределённость . Замена переменной уже напрашивается, но сначала преобразуем тангенс по формуле . Действительно, зачем нам тангенс?

Заметьте, что , поэтому . Если не совсем понятно, посмотрите значения синуса в тригонометрической таблице . Таким образом, мы сразу избавляемся от множителя , кроме того, получаем более привычную неопределённость 0:0. Хорошо бы ещё и предел у нас стремился к нулю.

Проведем замену:

Если , то

Под косинусом у нас находится «икс», который тоже необходимо выразить через «тэ».
Из замены выражаем: .

Завершаем решение:

(1) Проводим подстановку

(2) Раскрываем скобки под косинусом.

(4) Чтобы организовать первый замечательный предел , искусственно домножаем числитель на и обратное число .

Задание для самостоятельного решения:

Пример 17

Найти предел

Полное решение и ответ в конце урока.

Это были несложные задачи в своём классе, на практике всё бывает хуже, и, помимо формул приведения , приходится использовать самые разные тригонометрические формулы , а также прочие ухищрения. В статье Сложные пределы я разобрал пару настоящих примеров =)

В канун праздника окончательно проясним ситуацию ещё с одной распространённой неопределённостью:

Устранение неопределённости «единица в степени бесконечность»

Данную неопределённость «обслуживает» второй замечательный предел , и во второй части того урока мы очень подробно рассмотрели стандартные примеры решений, которые в большинстве случаев встречаются на практике. Сейчас картина с экспонентами будет завершена, кроме того, заключительные задания урока будут посвящены пределам-«обманкам», в которых КАЖЕТСЯ, что необходимо применить 2-й замечательный предел, хотя это вовсе не так.

Недостаток двух рабочих формул 2-го замечательного предела состоит в том, что аргумент должен стремиться к «плюс бесконечности» либо к нулю. Но что делать, если аргумент стремится к другому числу?

На помощь приходит универсальная формула (которая на самом деле является следствием второго замечательного предела):

Неопределённость можно устранить по формуле:

Где-то вроде уже пояснял, что обозначают квадратные скобки. Ничего особенного, скобки как скобки. Обычно их используют, чтобы чётче выделить математическую запись.

Выделим существенные моменты формулы:

1) Речь идёт только о неопределённости и никакой другой .

2) Аргумент «икс» может стремиться к произвольному значению (а не только к нулю или ), в частности, к «минус бесконечности» либо к любому конечному числу.

С помощью данной формулы можно решить все примеры урока Замечательные пределы , которые относятся ко 2-му замечательному пределу. Например, вычислим предел :

В данном случае , и по формуле :

Правда, делать так не советую, в традициях всё-таки применять «обычное» оформление решения, если его можно применить. 2 стремится к нулю.

Обычно переменная величина x стремится к конечному пределу a, причем, x постоянно приближается к a, а величина a постоянна. Это записывают следующим образом: limx =a, при этом, n также может стремиться как к нулю, так и к бесконечности. Существуют бесконечные функции, для них предел стремится к бесконечности. В других случаях, когда, например, функцией замедление хода поезда, можно о пределе, стремящемся к нулю.
У пределов имеется ряд свойств. Как правило, любая функция имеет только один предел. Это главное свойство предела. Другие их свойства перечислены ниже:
* Предел суммы равен сумме пределов:
lim(x+y)=lim x+lim y
* Предел произведения равен произведению пределов:
lim(xy)=lim x*lim y
* Предел частного равен частному от пределов:
lim(x/y)=lim x/lim y
* Постоянный множитель выносят за знак предела:
lim(Cx)=C lim x
Если дана функция 1 /x, в которой x →∞, ее предел равен нулю. Если же x→0, предел такой функции равен ∞.
Для тригонометрических функций имеются исключения из этих правил. Так как функция sin x всегда стремится к единице, когда приближается к нулю, для нее справедливо тождество:
lim sin x/x=1

В ряде задач встречаются функции, при вычислении пределов которых возникает неопределенность — ситуация, при которой предел невозможно вычислить. Единственным выходом из такой ситуации становится применение правила Лопиталя. Существует два вида неопределенностей:
* неопределенность вида 0/0
* неопределенность вида ∞/∞
К примеру, дан предел следующего вида: lim f(x)/l(x), причем, f(x0)=l(x0)=0. В таком случае, возникает неопределенность вида 0/0. Для решения такой задачи обе функции подвергают дифференцированию, после чего находят предел результата. Для неопределенностей вида 0/0 предел равен:
lim f(x)/l(x)=lim f»(x)/l»(x) (при x→0)
Это же правило справедливо и для неопределенностей типа ∞/∞. Но в этом случае справедливо следующее равенство: f(x)=l(x)=∞
С помощью правила Лопиталя можно находить значения любых пределов, в которых фигурируют неопределенности. (n-1)

Пределы доставляют всем студентам, изучающим математику, немало хлопот. Чтобы решить предел, порой приходится применять массу хитростей и выбирать из множества способов решения именно тот, который подойдет для конкретного примера.

В этой статье мы не поможем вам понять пределы своих возможностей или постичь пределы контроля, но постараемся ответить на вопрос: как понять пределы в высшей математике? Понимание приходит с опытом, поэтому заодно приведем несколько подробных примеров решения пределов с пояснениями.

Понятие предела в математике

Первый вопрос: что это вообще за предел и предел чего? Можно говорить о пределах числовых последовательностей и функций. Нас интересует понятие предела функции, так как именно с ними чаще всего сталкиваются студенты. Но сначала — самое общее определение предела:

Допустим, есть некоторая переменная величина. Если эта величина в процессе изменения неограниченно приближается к определенному числу a , то a – предел этой величины.

Для определенной в некотором интервале функции f(x)=y пределом называется такое число A , к которому стремится функция при х , стремящемся к определенной точке а . Точка а принадлежит интервалу, на котором определена функция.

Звучит громоздко, но записывается очень просто:

Lim — от английского limit — предел.

Существует также геометрическое объяснение определения предела, но здесь мы не будем лезть в теорию, так как нас больше интересует практическая, нежели теоретическая сторона вопроса. Когда мы говорим, что х стремится к какому-то значению, это значит, что переменная не принимает значение числа, но бесконечно близко к нему приближается.

Приведем конкретный пример. Задача — найти предел.

Чтобы решить такой пример, подставим значение x=3 в функцию. Получим:

Кстати, если Вас интересуют базовые операции над матрицами , читайте отдельную статью на эту тему.

В примерах х может стремиться к любому значению. Это может быть любое число или бесконечность. Вот пример, когда х стремится к бесконечности:

Интуитивно понятно, что чем больше число в знаменателе, тем меньшее значение будет принимать функция. Так, при неограниченном росте х значение 1/х будет уменьшаться и приближаться к нулю.

Как видим, чтобы решить предел, нужно просто подставить в функцию значение, к которому стремиться х . Однако это самый простой случай. Часто нахождение предела не так очевидно. В пределах встречаются неопределенности типа 0/0 или бесконечность/бесконечность . Что делать в таких случаях? Прибегать к хитростям!


Неопределенности в пределах

Неопределенность вида бесконечность/бесконечность

Пусть есть предел:

Если мы попробуем в функцию подставить бесконечность, то получим бесконечность как в числителе, так и в знаменателе. Вообще стоит сказать, что в разрешении таких неопределенностей есть определенный элемент искусства: нужно заметить, как можно преобразовать функцию таким образом, чтобы неопределенность ушла. В нашем случае разделим числитель и знаменатель на х в старшей степени. Что получится?

Из уже рассмотренного выше примера мы знаем, что члены, содержащие в знаменателе х, будут стремиться к нулю. Тогда решение предела:

Для раскрытия неопределенностей типа бесконечность/бесконечность делим числитель и знаменатель на х в высшей степени.


Кстати! Для наших читателей сейчас действует скидка 10% на любой вид работы

Еще один вид неопределенностей: 0/0

Как всегда, подстановка в функцию значения х=-1 дает 0 в числителе и знаменателе. Посмотрите чуть внимательнее и Вы заметите, что в числителе у нас квадратное уравнение. Найдем корни и запишем:

Сократим и получим:

Итак, если Вы сталкиваетесь с неопределенностью типа 0/0 – раскладывайте числитель и знаменатель на множители.

Чтобы Вам было проще решать примеры, приведем таблицу с пределами некоторых функций:

Правило Лопиталя в пределах

Еще один мощный способ, позволяющий устранить неопределенности обоих типов. В чем суть метода?

Если в пределе есть неопределенность, берем производную от числителя и знаменателя до тех пор, пока неопределенность не исчезнет.

Наглядно правило Лопиталя выглядит так:

Важный момент : предел, в котором вместо числителя и знаменателя стоят производные от числителя и знаменателя, должен существовать.

А теперь – реальный пример:

Налицо типичная неопределенность 0/0 . Возьмем производные от числителя и знаменателя:

Вуаля, неопределенность устранена быстро и элегантно.

Надеемся, что Вы сможете с пользой применить эту информацию на практике и найти ответ на вопрос «как решать пределы в высшей математике». Если нужно вычислить предел последовательности или предел функции в точке, а времени на эту работу нет от слова «совсем», обратитесь в профессиональный студенческий сервис за быстрым и подробным решением.

формула, теорема, как найти предел

Мы уже начали разбираться с пределами и их решением. Продолжим по горячим следам и разберемся с решением пределов по правилу Лопиталя. Этому простому правилу по силам помочь Вам выбраться из коварных и сложных ловушек, которые преподаватели так любят использовать в примерах на контрольных по высшей математике и матанализу. Решение правилом Лопиталя – простое и быстрое. Главное – уметь дифференцировать.

Правило Лопиталя: история и определение

На самом деле это не совсем правило Лопиталя, а правило Лопиталя-Бернулли. Сформулировал его швейцарский математик Иоганн Бернулли, а француз Гийом Лопиталь впервые опубликовал в своем учебнике бесконечно малых в славном 1696 году. Представляете, как людям приходилось решать пределы с раскрытием неопределенностей до того, как это случилось? Мы – нет.

Кстати, о том, какой вклад внес в науку сын Иоганна Бернулли, читайте в статье про течение жидкостей и уравнение Бернулли.

Пределы

Прежде чем приступать к разбору правила Лопиталя, рекомендуем прочитать вводную статью про пределы в математике и методы их решений. Часто в заданиях встречается формулировка: найти предел, не используя правило Лопиталя. О приемах, которые помогут Вам в этом, также читайте в нашей статье.

Если имеешь дело с пределами дроби двух функций, будь готов: скоро встретишься с неопределенностью вида 0/0 или бесконечность/бесконечность. Как это понимать? В числителе и знаменателе выражения стремятся к нулю или бесконечности. Что делать с таким пределом, на первый взгляд – совершенно непонятно. Однако если применить правило Лопиталя и немного подумать, все становится на свои места.

Но сформулируем правило Лопиталя-Бернулли. Если быть совершенно точными, оно выражается теоремой. Правило Лопиталя, определение:

Если две функции дифференцируемы в окрестности точки x=a обращаются в нуль в этой точке, и существует предел отношения производных этих функций, то при х стремящемся к а существует предел отношения самих функций, равный пределу отношения производных.

Запишем формулу, и все сразу станет проще. Правило Лопиталя, формула:

Так как нас интересует практическая сторона вопроса, не будем приводить здесь доказательство этой теоремы. Вам придется или поверить нам на слово, или найти его в любом учебнике по математическому анализу и убедится, что теорема верна.

Кстати! Для наших читателей сейчас действует скидка 10% на любой вид работы

Раскрытие неопределенностей по правилу Лопиталя

В раскрытии каких неопределенностей может помочь правило Лопиталя? Ранее мы говорили в основном о неопределенности 0/0. Однако это далеко не единственная неопределенность, с которой можно встретиться. Вот другие виды неопределенностей:

Рассмотрим преобразования, с помощью которых можно привести эти неопределенности к виду 0/0 или бесконечность/бесконечность. После преобразования можно будет применять правило Лопиталя-Бернулли и щелкать примеры как орешки.

Неопределенности

Неопределенность вида бесконечность/бесконечность сводится к неопределенность вида 0/0 простым преобразованием:

Пусть есть произведение двух функций, одна из которых первая стремиться к нулю, а вторая – к бесконечности. Применяем преобразование, и произведение нуля и бесконечности превращается в неопределенность 0/0:

Для нахождения пределов с неопределенностями типа бесконечность минус бесконечность используем следующее преобразование,  приводящее к неопределенности 0/0:

Для того чтобы пользоваться правилом Лопиталя, нужно уметь брать производные. Приведем ниже таблицу производных элементарных функций, которой Вы сможете пользоваться при решении примеров, а также правила вычисления производных сложных функций:

Таблица производных

Теперь перейдем к примерам.

Пример 1

Найти предел по правилу Лопиталя:

Пример 2

Вычислить с использованием правила Лопиталя:

Важный момент! Если предел вторых и последующих производных функций существует при х стремящемся к а, то правило Лопиталя можно применять несколько раз.

Найдем предел (n – натуральное число). Для этого применим правило Лопиталя n раз:

Желаем удачи в освоении математического анализа. А если Вам понадобится найти предел используя правило Лопиталя, написать реферат по правилу Лопиталя, вычислить корни дифференциального уравнения или даже рассчитать тензор инерции тела, обращайтесь к нашим авторам. Они с радостью помогут разобраться в тонкостях решения.

Предел функции простым языком — Altarena.ru — технологии и ответы на вопросы

Содержание

  1. Предел функции: основные понятия и определения
  2. Понятие предела
  3. Что такое предел функции
  4. Пределы
  5. Что такое пределы простыми словами
  6. Пределы в математике
  7. Еще один пример
  8. Предел последовательности
  9. Можно сказать и так:
  10. Зачем нужны пределы
  11. Примеры из жизни
  12. Пределы функций. Примеры решений
  13. Видео

Предел функции: основные понятия и определения

В этой статье мы расскажем, что из себя представляет предел функции. Сначала поясним общие моменты, которые очень важны для понимания сути этого явления.

Понятие предела

Если мы не можем определить ни конечное, ни бесконечное значение, это значит, что такого предела не существует. Примером этого случая может быть предел от синуса на бесконечности.

Что такое предел функции

В этом пункте мы объясним, как найти значение предела функции в точке и на бесконечности. Для этого нам нужно ввести основные определения и вспомнить, что такое числовые последовательности, а также их сходимость и расходимость.

При x → ∞ предел функции f ( x ) является бесконечным, если последовательность значений для любой бесконечно большой последовательности аргументов будет также бесконечно большой (положительной или отрицательной).

Решение

Далее мы запишем то же самое, но для бесконечно большой отрицательной последовательности.

Здесь тоже видно монотонное убывание к нулю, что подтверждает верность данного в условии равенства:

Ответ: Верность данного в условии равенства подтверждена.

Решение

Мы видим, что данная последовательность бесконечно положительна, значит, f ( x ) = lim x → + ∞ e 1 10 x = + ∞

Наглядно решение задачи показано на иллюстрации. Синими точками отмечена последовательность положительных значений, зелеными ­ – отрицательных.

Перейдем к методу вычисления предела функции в точке. Для этого нам нужно знать, как правильно определить односторонний предел. Это пригодится нам и для того, чтобы найти вертикальные асимптоты графика функции.

Теперь сформулируем, что такое предел функции справа.

Теперь мы разъясним данные определения, записав решение конкретной задачи.

Решение

Значения функции в этой последовательности будут выглядеть так:

Чтобы более глубоко изучить теорию пределов, советуем вам прочесть статью о непрерывности функции в точке и основных видах точек разрыва.

Источник

Пределы

Пределы — одни из самых трудных сущностей в математике для понимания. Сложно объяснить просто, что такое предел, поэтому чаще всего этого никто и не делает.

И тем более, мало к то из преподавателей может привести пример из жизни, когда пределы все-таки могут пригодится. Но мы попытаемся объяснить так, чтобы было и понятно и несложно и по сути. Как обычно «на пальцах».

Что такое пределы простыми словами

Наверное самое наглядное, что можно вспомнить из истории, это знаменитый парадокс Зенона «Ахиллес и черепаха». Зенон был философом, а не математиком, поэтому мог вполне свободно упражняется в остроумии не заботясь о доказательствах.

Ахиллес и черепаха бегут на перегонки. Черепаха начинает первой, человек догоняет. Ахиллес бежит быстрее, но когда он пробегает 100 шагов, черепаха все рано проползает один. Еще 100 шагов и еще один. Таким образом Ахиллес приближается к черепахе но и она чуть-чуть отдаляется от него. Зенон делает вывод, что Ахиллес будет бесконечно к ней приближаться, но никогда не догонит черепаху!

В этой истории важно не то, что на самом деле она не реальна, а ее «математический смысл». Человек приближается к черепахе но никогда ее не настигает. То есть некий предел (черепаха) к которому стремится Ахиллес.

Говоря простым языком предел это такое значение, которое нельзя достичь, но можно бесконечно близко к нему приблизится.

То есть, в пределе определенного промежутка времени Ахиллес действительно не догонит черепаху (времени не хватит), но приблизится к ней на бесконечно малое расстояние.

Пределы в математике

Стоит сразу сказать, что определение пределов больше чем одно, потому, что они бывают разные. Есть придел последовательности, а есть предел функции.

Давайте разделим число 10 пополам:

10/2=5, и еще раз, 5/2=2,5 и еще…

Это последовательность n/2: 10…2,5…1,25…

Если делать это 20 раз получится вот такое значение: 0,000019

А если сделать 100 раз, то вот такое: 0,000000000000000000000000000016

Если делить пополам бесконечно, результат будет уменьшатся, в реальной жизни, это будет уже фактически ноль, но в математике, все еще не ноль… Предел этой последовательности будет стремиться к нолю.

Если взять другу последовательность, например n+1. 2…3…4…5… и снова устремимся в бесконечность. Предел этого множества тоже будет стремится к бесконечности.

Еще один пример

Бросаем монетку. Может выпасть «орел», а может и «решка». Теория вероятности утверждает, что шансы всегда 50/50, то есть вероятность «орла» — 1/2=0,5.

Каждый раз, значение реально вероятности, приближается к расчетным 0,5. Чтобы получить вероятность ровно 0,5 нужно подбросить монетку бесконечное количество раз.

То есть, при условии, что количество бросков стремится к бесконечности предел предел будет равен 0,5.

Это именно та бесконечность из матанализа о которой было сказано в статьях об интегралах и делении на ноль. Это не какое-то определенное число — это понятие.

Предел последовательности

Предел последовательности — это пространство которое содержит все все элементы последовательности начиная с какого-то значения. А простыми словами, предел последовательности, простыми словами, это такая «область» куда попадают все значения после определенного порога (в нашем случае – А). На изображении ниже она условно показана синей полоской.

ε — это произвольное положительное число.

Можно заметить, что при продолжении вверх последовательности ее значения все равно будут оставаться в пределах «синей полосы».

Можно сказать и так:

Предел числовой последовательности, это число (s на графике) в окрестности которого попадает бесконечно много значений. При этом вне предела, количество значений явно конечно. Чтобы было еще понятнее: предел последовательности это значение (точка А) выше которого все будет попадать в область не больше s+ε и s-ε. Бесконечное количество таких значений будет «лежать» внутри синей полоски.

Математическим языком можно записать так: s-ε Предел функции простыми словами объяснить также просто. Предел в какой-то произвольной точке — это величина к которой значение функции приближается. Например, f(x)=2x, а х→0 (икс стремится к нулю).

В этом случае предел функции будет равен lim 2x=0. Или в случае если х→2 то предел равен lim 2x=4. Пока все просто. Вот только зачем вычислять пределы, если можно просто выбросить «lim» и расчеты останутся те ми же?….

Зачем нужны пределы

Пределы как раз и нужны тогда, когда мы имеем дело с бесконечностью. Например, бесконечно большими или бесконечно малыми значениями.

Непонятно, что такое «бесконечно большое» или «бесконечно долго», это не какое-то определенное число. С бесконечно малыми значениями та же ситуация, это не «ноль» но как-то очень близко к нему. Тут и выручают пределы.

В точке х=2 — пусто. Потому, что получается 0/0, то есть неопределенность. Но стоит вместо 2 подставить 1,9999999999(9) или 2,000000001(1). Значения бесконечно близкие к 2, но не «два», как график превратится в прямую.

В этом случае речь идет о пределе функции при «икс» стремящемуся к двум, функция стремится к 4.

Такой своеобразный «трюк» в расчетах с заменой знака равенства на стрелочку.

Нет, не совсем. Когда речь идет о пределах, имеется в виду процесс, не важно функция это или множество, но предел описывает процесс в динамике. Тогда как знак «равно» означает статическое состояние.

x=1 и x→1, это совсем не одно и то же.

Примеры из жизни

Зачем все это нужно где применяется пределы в реальных расчетах?

Простое объяснение пределов невозможно, если не привести наглядный пример. Но только где его взять? Существует ли какой-то физический смысл пределов? Не точный аналог но что-то похожее есть.

Можно провести простой эксперимент, взять, например, спичку. Или что-угодно, чего не жалко. Начинаем пытаться сломать спичку, сначала одно усилие, потом чуть больше и еще больше. В один из моментов спичка треснет пополам.

Поздравляем, вы достигли предела прочности. Можно повторить эксперимент с другими спичками и установить, значение при котором спичка ломается.

Что тут общего с пределами из математики, кроме названия.

Есть множество значений силы до предела прочности и оно ограничено, и множество значений после предела прочности, их неограниченное множество. Ведь спичка уже сломана, любое усилие выше предела прочности будет ломать новую и новую спичку. Точно так же как и с пределом функции или множества.

Все, что лежит за пределом, уже не имеет практического значения — спичка не устоит.

Еще один пример, это «практический потолок» летательного аппарата. Это максимальная высота на которую может «взобраться» самолет, чтобы подняться выше будет уже не хватать подъемной силы. Хотя на есть еще и понятие «динамический потолок» — это высота на которую можно подняться хорошенько разогнавшись. Но выскочив на эту высоту через некоторое время самолет все равно опустится на свой «потолок».

Посмотрите на картинку ниже, это наглядный пример такого явления как резонанс.

Колебание моста из-за резонанса

Мост так раскачивается из-за того, что собственная частота колебания совпадает с той частотой с которой его раскачивает ветер, амплитуда колебаний постоянно возрастает и мост разрушается. В этом случае амплитуда стремится к бесконечности, так как в знаменателе формулы находится выражение w0-w (собственная частота колебаний минус вынужденная частота), а так как обе w равны, получается то самое деление на ноль, а значит амплитуда → ∞.

Самое понятное объяснений пределов в реальности, с которым может столкнуться каждый — это сложные банковские проценты по кредиту. И если вы не умеете рассчитывать сложны проценты, не берите кредит. Для тех, кто силен в матанализе совет будет не лишним.

Также может понадобится рассчитать предельную стоимость товара, зная зависимость (функцию) цены от объема продаж или предельный объем производства или много еще чего.

Самый наглядный пример, возможно, это предел в маркетинге. Вот зависимость стоимости клика от количества кликов в контекстной рекламе.

И все же в повседневной жизни обыватель редко встречается с таким понятием как предел функции или последовательности. Поэтому и так сложно понять и принять абстрактные математические формулировки. Но если постараться, математика может открыть новые грани реальности, по крайней мере, все это уже не будет казаться таким скучным и непонятным.

Источник

Пределы функций. Примеры решений

Теория пределов – это один из разделов математического анализа. Вопрос решения пределов является достаточно обширным, поскольку существуют десятки приемов решений пределов различных видов. Существуют десятки нюансов и хитростей, позволяющих решить тот или иной предел. Тем не менее, мы все-таки попробуем разобраться в основных типах пределов, которые наиболее часто встречаются на практике.

Начнем с самого понятия предела. Но сначала краткая историческая справка. Жил-был в 19 веке француз Огюстен Луи Коши, который дал строгие определения многим понятиям матана и заложил его основы. Надо сказать, этот уважаемый математик снился, снится и будет сниться в кошмарных снах всем студентам физико-математических факультетов, так как доказал огромное количество теорем математического анализа, причём одна теорема убойнее другой. В этой связи мы пока не будем рассматривать определение предела по Коши, а попытаемся сделать две вещи:

1. Понять, что такое предел.
2. Научиться решать основные типы пределов.

Прошу прощения за некоторую ненаучность объяснений, важно чтобы материал был понятен даже чайнику, что, собственно, и является задачей проекта.

Итак, что же такое предел?

А сразу пример, чего бабушку лохматить….

Любой предел состоит из трех частей:

1) Всем известного значка предела .
2) Записи под значком предела, в данном случае . Запись читается «икс стремится к единице». Чаще всего – именно , хотя вместо «икса» на практике встречаются и другие переменные. В практических заданиях на месте единицы может находиться совершенно любое число, а также бесконечность ().
3) Функции под знаком предела, в данном случае .

Сама запись читается так: «предел функции при икс стремящемся к единице».

Разберем следующий важный вопрос – а что значит выражение «икс стремится к единице»? И что вообще такое «стремится»?
Понятие предела – это понятие, если так можно сказать, динамическое. Построим последовательность: сначала , затем , , …, , ….
То есть выражение «икс стремится к единице» следует понимать так – «икс» последовательно принимает значения, которые бесконечно близко приближаются к единице и практически с ней совпадают.

Как решить вышерассмотренный пример? Исходя из вышесказанного, нужно просто подставить единицу в функцию, стоящую под знаком предела:

Итак, первое правило: Когда дан любой предел, сначала просто пытаемся подставить число в функцию.

Мы рассмотрели простейший предел, но и такие встречаются на практике, причем, не так уж редко!

Пример с бесконечностью:

Разбираемся, что такое ? Это тот случай, когда неограниченно возрастает, то есть: сначала , потом , потом , затем и так далее до бесконечности.

А что в это время происходит с функцией ?
, , , …

Итак: если , то функция стремится к минус бесконечности:

Грубо говоря, согласно нашему первому правилу, мы вместо «икса» подставляем в функцию бесконечность и получаем ответ.

Еще один пример с бесконечностью:

Опять начинаем увеличивать до бесконечности и смотрим на поведение функции:

Вывод: при функция неограниченно возрастает:

И еще серия примеров:

Пожалуйста, попытайтесь самостоятельно мысленно проанализировать нижеследующее и запомните простейшие виды пределов:

, , , , , , , , ,
Если где-нибудь есть сомнения, то можете взять в руки калькулятор и немного потренироваться.
В том случае, если , попробуйте построить последовательность , , . Если , то , , .

! Примечание: строго говоря, такой подход с построением последовательностей из нескольких чисел некорректен, но для понимания простейших примеров вполне подойдет.

Также обратите внимание на следующую вещь. Даже если дан предел с большим числом вверху, да хоть с миллионом: , то все равно , так как рано или поздно «икс» начнёт принимать такие гигантские значения, что миллион по сравнению с ними будет самым настоящим микробом.

Что нужно запомнить и понять из вышесказанного?

1) Когда дан любой предел, сначала просто пытаемся подставить число в функцию.

2) Вы должны понимать и сразу решать простейшие пределы, такие как , , и т.д.

Более того, у предела есть очень хороший геометрический смысл. Для лучшего понимания темы рекомендую ознакомиться с методическим материалом Графики и свойства элементарных функций. После прочтения этой статьи вы не только окончательно поймете, что такое предел, но и познакомитесь с интересными случаями, когда предела функции вообще не существует!

На практике, к сожалению, подарков немного. А поэтому переходим к рассмотрению более сложных пределов. Кстати, по этой теме есть интенсивный курс в pdf-формате, который особенно полезен, если у Вас ОЧЕНЬ мало времени на подготовку. Но материалы сайта, разумеется, не хуже:

Пределы с неопределенностью вида и метод их решения

Сейчас мы рассмотрим группу пределов, когда , а функция представляет собой дробь, в числителе и знаменателе которой находятся многочлены

Вычислить предел

Согласно нашему правилу попытаемся подставить бесконечность в функцию. Что у нас получается вверху? Бесконечность. А что получается внизу? Тоже бесконечность. Таким образом, у нас есть так называемая неопределенность вида . Можно было бы подумать, что , и ответ готов, но в общем случае это вовсе не так, и нужно применить некоторый прием решения, который мы сейчас и рассмотрим.

Как решать пределы данного типа?

Сначала мы смотрим на числитель и находим в старшей степени:

Старшая степень в числителе равна двум.

Теперь смотрим на знаменатель и тоже находим в старшей степени:

Старшая степень знаменателя равна двум.

Затем мы выбираем самую старшую степень числителя и знаменателя: в данном примере они совпадают и равны двойке.

Итак, метод решения следующий: для того, чтобы раскрыть неопределенность необходимо разделить числитель и знаменатель на в старшей степени.


Разделим числитель и знаменатель на

Вот оно как, ответ , а вовсе не бесконечность.

Что принципиально важно в оформлении решения?

Во-первых, указываем неопределенность, если она есть.

Во-вторых, желательно прервать решение для промежуточных объяснений. Я обычно использую знак , он не несет никакого математического смысла, а обозначает, что решение прервано для промежуточного объяснения.

В-третьих, в пределе желательно помечать, что и куда стремится. Когда работа оформляется от руки, удобнее это сделать так:

Для пометок лучше использовать простой карандаш.

Конечно, можно ничего этого не делать, но тогда, возможно, преподаватель отметит недочеты в решении либо начнет задавать дополнительные вопросы по заданию. А оно Вам надо?

Найти предел
Снова в числителе и знаменателе находим в старшей степени:

Максимальная степень в числителе: 3
Максимальная степень в знаменателе: 4
Выбираем наибольшее значение, в данном случае четверку.
Согласно нашему алгоритму, для раскрытия неопределенности делим числитель и знаменатель на .
Полное оформление задания может выглядеть так:

Разделим числитель и знаменатель на

Найти предел
Максимальная степень «икса» в числителе: 2
Максимальная степень «икса» в знаменателе: 1 ( можно записать как )
Для раскрытия неопределенности необходимо разделить числитель и знаменатель на . Чистовой вариант решения может выглядеть так:

Разделим числитель и знаменатель на

Под записью подразумевается не деление на ноль (делить на ноль нельзя), а деление на бесконечно малое число.

Таким образом, при раскрытии неопределенности вида у нас может получиться конечное число, ноль или бесконечность.

Пределы с неопределенностью вида и метод их решения

Предвосхищаю вопрос от чайников: «Почему здесь деление на ноль? На ноль же делить нельзя!». Смысл записи 0:0 будет понятен позже, после ознакомления с четвёртым уроком о бесконечно малых функциях. А пока всем начинающим изучать математический анализ предлагаю читать далее.

Следующая группа пределов чем-то похожа на только что рассмотренные пределы: в числителе и знаменателе находятся многочлены, но «икс» стремится уже не к бесконечности, а к конечному числу.

Общее правило: если в числителе и знаменателе находятся многочлены, и имеется неопределенности вида , то для ее раскрытия нужно разложить числитель и знаменатель на множители.

Для этого чаще всего нужно решить квадратное уравнение и (или) использовать формулы сокращенного умножения. Если данные вещи позабылись, тогда посетите страницу Математические формулы и таблицы и ознакомьтесь с методическим материалом Горячие формулы школьного курса математики. Кстати его лучше всего распечатать, требуется очень часто, да и информация с бумаги усваивается лучше.

Итак, решаем наш предел

Разложим числитель и знаменатель на множители

Для того чтобы разложить числитель на множители, нужно решить квадратное уравнение:

Сначала находим дискриминант:

И квадратный корень из него: .

В случае если дискриминант большой, например 361, используем калькулятор, функция извлечения квадратного корня есть на самом простом калькуляторе.

! Если корень не извлекается нацело (получается дробное число с запятой), очень вероятно, что дискриминант вычислен неверно либо в задании опечатка.

Далее находим корни:

Таким образом:

Всё. Числитель на множители разложен.

Знаменатель. Знаменатель уже является простейшим множителем, и упростить его никак нельзя.

Очевидно, что можно сократить на :

Естественно, в контрольной работе, на зачете, экзамене так подробно решение никогда не расписывают. В чистовом варианте оформление должно выглядеть примерно так:

Разложим числитель на множители.





Вычислить предел

Сначала «чистовой» вариант решения

Разложим числитель и знаменатель на множители.

Числитель:
Знаменатель:



,

Что важного в данном примере?
Во-первых, Вы должны хорошо понимать, как раскрыт числитель, сначала мы вынесли за скобку 2, а затем использовали формулу разности квадратов. Уж эту-то формулу нужно знать и видеть.

Рекомендация: Если в пределе (практически любого типа) можно вынести число за скобку, то всегда это делаем.
Более того, такие числа целесообразно выносить за значок предела. Зачем? Да просто чтобы они не мешались под ногами. Главное, потом эти числа не потерять по ходу решения.

Обратите внимание, что на заключительном этапе решения я вынес за значок предела двойку, а затем – минус.

Вообще, я заметил, что чаще всего в нахождении пределов данного типа приходится решать два квадратных уравнения, то есть и в числителе и в знаменателе находятся квадратные трехчлены.

Метод умножения числителя и знаменателя на сопряженное выражение

Продолжаем рассматривать неопределенность вида

Следующий тип пределов похож на предыдущий тип. Единственное, помимо многочленов, у нас добавятся корни.

Найти предел

Сначала пробуем подставить 3 в выражение под знаком предела
Еще раз повторяю – это первое, что нужно выполнять для ЛЮБОГО предела. Данное действие обычно проводится мысленно или на черновике.

Получена неопределенность вида , которую нужно устранять.

Как Вы, наверное, заметили, у нас в числителе находится разность корней. А от корней в математике принято, по возможности, избавляться. Зачем? А без них жизнь проще.

Когда в числителе (знаменателе) находится разность корней (или корень минус какое-нибудь число), то для раскрытия неопределенности используют метод умножения числителя и знаменателя на сопряженное выражение.

Вспоминаем нашу нетленную формулу разности квадратов:
И смотрим на наш предел:
Что можно сказать? у нас в числителе уже есть. Теперь для применения формулы осталось организовать (которое и называется сопряженным выражением).

Умножаем числитель на сопряженное выражение:

Обратите внимание, что под корнями при этой операции мы ничего не трогаем.

Хорошо, мы организовали, но выражение-то под знаком предела изменилось! А для того, чтобы оно не менялось, нужно его разделить на то же самое, т.е. на :

То есть, мы умножили числитель и знаменатель на сопряженное выражение.
В известной степени, это искусственный прием.

Умножили. Теперь самое время применить вверху формулу :

Неопределенность не пропала (попробуйте подставить тройку), да и корни тоже не исчезли. Но с суммой корней всё значительно проще, ее можно превратить в постоянное число. Как это сделать? Да просто подставить тройку под корни:

Число, как уже отмечалось ранее, лучше вынести за значок предела.

Теперь осталось разложить числитель и знаменатель на множители и сократить «виновников» неопределённости, ну а предел константы – равен самой константе:

Как должно выглядеть решение данного примера в чистовом варианте?
Примерно так:

Умножим числитель и знаменатель на сопряженное выражение.

Найти предел

Сначала попробуйте решить его самостоятельно.

Окончательное решение примера может выглядеть так:

Разложим числитель на множители:





Умножим числитель и знаменатель на сопряженное выражение

Спасибо за внимание.

Помимо рассмотренных типов пределов на практике часто встречаются так называемые Замечательные пределы. После освоения двух базовых уроков, рекомендую изучить статью Методы решения пределов, материалы которой позволят выйти на «твёрдую четвёрку»!

Автор: Емелин Александр

(Переход на главную страницу)

«Всё сдал!» — онлайн-сервис помощи студентам

Источник

Видео

Пределы функций для чайников. Свойства пределов. Примеры решения

ПРЕДЕЛ ФУНКЦИИ. Артур Шарифов

Матан. Пределы для успешной сдачи зачёта | TutorOnline Математика

Что такое ПРЕДЕЛЫ. Математика на QWERTY

Математика без Ху{a59d68d9e85770d26d7f03ef1e17421f46fa783a9a8ee536de468ee46886eb77}!ни. Пределы, часть1. Неопределенность, раскрытие неопределенностей.

10 класс, 39 урок, Предел функции

ПРЕДЕЛ ФУНКЦИИ решение пределов математика

27. Вычисление предела функции №1. Примеры 1-4

✓ Предел функции. Определение предела функции «по Коши» и «по Гейне» | матан #014 | Борис Трушин

Предел функции в точке. 10 класс.

Правила вычисления пределов — Мегаобучалка

 

При вычислении пределов следует учитывать следующие основные правила:

 

1. Предел суммы (разности) функций равен сумме (разности) пределов слагаемых:

.

 

2. Предел произведения функций равен произведению пределов сомножителей:

.

 

3. Предел отношения двух функций равен отношению пределов этих функций:

.

 

4. Постоянный множитель можно выносить за знак предела:

.

 

5. Предел постоянной равен самой постоянной:

 

.

6. Для непрерывных функций символы предела и функции можно поменять местами:

 

.

 

Нахождение предела функции следует начинать с подстановки значения в выражение для функции. При этом если получается числовое значение 0 или ¥, то искомый предел найден.

Пример 2.1.Вычислить предел .

Решение.

.

Выражения вида , , , , , называются неопределённостями.

Если получается неопределенность вида , то для нахождения предела нужно преобразовать функцию так, чтобы раскрыть эту неопределенность.

Неопределенность вида обычно получается, когда задан предел отношения двух многочленов. В этом случае, для вычисления предела рекомендуется разложить многочлены на множители и сократить на общий множитель. Этот множитель равен нулю при предельном значении х.



Пример 2.2.Вычислить предел .

Решение.

Подставляя , получим неопределенность:

 

.

 

Разложим числитель и знаменатель на множители:

;

 

Сократим на общий множитель и получим

 

.

Неопределенность вида получается, когда задан предел отношения двух многочленов при . В этом случае для вычисления рекомендуется разделить оба многочлена на х в старшей степени.

Пример 2.3. Вычислить предел .

Решение.При подстановке ∞ получается неопределенность вида , поэтому разделим все члены выражения на x3.

.

Здесь учитывается, что .

При вычислении пределов функции, содержащей корни, рекомендуется умножить и разделить функцию на сопряженное выражение.

Пример 2.4.Вычислить предел

Решение.

 

При вычислении пределов для раскрытия неопределенности вида или (1) часто используются первый и второй замечательные пределы:

и

Ко второму замечательному пределу приводят многие задачи, связанные с непрерывным ростом какой-либо величины.

Рассмотрим пример Я. И. Перельмана, дающий интерпретацию числа e в задаче о сложных процентах. В сбербанках процентные деньги присоединяются к основному капиталу ежегодно. Если присоединение совершается чаще, то капитал растет быстрее, так как в образовании процентов участвует большая сумма. Возьмем чисто теоретический, весьма упрощенный пример.

Пусть в банк положено 100 ден. ед. из расчета 100 % годовых. Если процентные деньги будут присоединены к основному капиталу лишь по истечении года, то к этому сроку 100 ден. ед. превратятся в 200 ден.ед.

Посмотрим теперь, во что превратятся 100 ден. ед., если процентные деньги присоединять к основному капиталу каждые полгода. По истечении полугодия 100 ден. ед. вырастут в 100 × 1,5 = 150, а еще через полгода — в 150 × 1,5 = 225 (ден. ед.). Если присоединение делать каждые 1/3 года, то по истечении года 100 ден. ед. превратятся в 100 × (1 +1/3)3 »237 (ден. ед.).

Будем учащать сроки присоединения процентных денег до 0,1 года, до 0,01 года, до 0,001 года и т.д. Тогда из 100 ден. ед. спустя год получится:

100 × (1 +1/10)10 » 259 (ден. ед.),

100 × (1+1/100)100 » 270 (ден. ед.),

100 × (1+1/1000)1000 » 271 (ден. ед.).

При безграничном сокращении сроков присоединения процентов наращенный капитал не растет беспредельно, а приближается к некоторому пределу, равному приблизительно 271. Более чем в 2,71 раз капитал, положенный под 100% годовых, увеличиться не может, даже если бы наросшие проценты присоединялись к капиталу каждую секунду, потому что

Пример 2.5.Вычислить предел функции

Решение.

Пример 2.6.Вычислить предел функции .

Решение.Подставляя получим неопределенность:

 

.

Используя тригонометрическую формулу, преобразуем числитель в произведение:

 

В результате получаем

Здесь учитывается второй замечательный предел .

Пример 2.7.Вычислить предел функции

Решение.

.

Для раскрытия неопределенности вида или можно использовать правило Лопиталя, которое основано на следующей теореме.

Теорема.Предел отношения двух бесконечно малых или бесконечно больших функций равен пределу отношения их производных

 

Заметим, что это правило можно применять несколько раз подряд.

Пример 2.8. Найти

Решение.При подстановке , имеем неопределенность вида . Применяя правило Лопиталя, получим

 

Непрерывность функции

 

Важным свойством функции является непрерывность.

Определение.Функция считается непрерывной, если малое изменение значения аргумента влечет за собой малое изменение значения функции.

Математически это записывается так: при

Под и понимается приращение переменных, то есть разность между последующим и предыдущим значениями: , (рисунок 2.3)

 

Рисунок 2.3 – Приращение переменных

Из определения функции , непрерывной в точке , следует, что . Это равенство означает выполнение трех условий:

1) функция определена в точке и ее окрестности функция ;

2) функция имеет предел при или, что равносильно, существуют и равны односторонние пределы и ;

3) предел функции при равен значению функции в точке .

Если нарушается хотя бы одно из этих условий, то точку называют точкой разрыва функции. Выделяют следующие типы точек разрыва.

1) Если в точке разрыва существуют односторонние конечные пределы функции, то называют точкой разрыва первого рода.

При этом если односторонние пределы совпадают, то называют точкой устранимого разрыва первого рода, если односторонние пределы не совпадают, то называют точкой конечного разрыва первого рода (или точкой скачка)

2) Если в точке хотя бы один из односторонних пределов функции не существует или бесконечен, то называют точкой разрыва второго рода.

Пример 2.9.Найти точки разрыва функции:

Решение.Для функции точка является подозрительной на разрыв, проверим это, найдем односторонние пределы

Следовательно, , значит — точка устранимого разрыва

Производная функции

 

методов оценки пределов | Что такое методы оценки пределов — примеры и решения

Теперь мы обсудим различные методы, используемые для получения пределов. Каждый метод будет сопровождаться несколькими примерами, иллюстрирующими этот метод.

(A)         ПРЯМАЯ ПОДСТАВКА

Это уже упоминалось в начале текущего раздела, где мы видели, что для непрерывной функции предел может быть получен прямой подстановкой.

95} — 4x + 3}}\end{align}\)

Мы видим, что при подстановке x = 1 и числитель, и знаменатель становятся равными 0.

Следовательно, ( x – 1 ) является множитель числителя и знаменателя (теорема о множителях)

Факторизация приводит к

(C) РАЦИОНАЛИЗАЦИЯ

В этом методе рационализация неопределенного выражения приводит к определенному. Следующие примеры развивают этот метод.

92}\,.\,\,\frac{1}{{\cos \,h}} \end{align}\]

Теперь это выражение содержит только пределы \(\mathop {\lim }\limits_{ x \to 0} \frac{{\sin x}}{x} = 1\) и \(\mathop {\lim }\limits_{x \to 0} \cos x = 1\)

Следовательно, окончательный результат: \(\frac{1}{{16}}\)

Теперь мы рассмотрим примеры, основанные на рассмотренных выше методах. Мы настоятельно рекомендуем вам сначала опробовать все эти примеры самостоятельно, прежде чем просматривать решения.

Алгебраическое нахождение пределов

Алгебраическое нахождение пределов

К концу этой лекции вы должны быть в состоянии распознать, какие неопределенные выражения являются определенными, а какие нет, и вы должны быть в состоянии использовать эти знания для решения предельных задач, переписывая их алгебраически, пока не получите определенную форму. В частности, вы должны уметь находить пределы на бесконечности и определять, когда пределов не существует (а когда их не существует, объяснять почему). Вы также должны уметь правильно использовать предельные обозначения.

Прежде чем мы начнем эту лекцию, мы хотим напомнить себе об определении алгебры, которое будет важно при алгебраическом расчете предельных задач:

Определение: undefined

Помните, что в алгебре мы иногда получаем выражения, которые undefined . Неопределенное выражение — это выражение, которое не имеет одного четкого значения. Например, если бы мы могли доказать, что выражение имеет два разных значения, то это выражение было бы неопределенным, потому что мы не допускаем, чтобы выражения были равны двум различным значениям. вещи сразу (потому что это привело бы к сумасшедшим противоречиям вроде 2=5!).

Другая причина, по которой выражение может быть неопределенным, заключается в том, что оно не определено по отношению к набору чисел, с которым мы сейчас работаем. Например, если мы работаем только с набором действительных чисел, любое выражение, которое дает нам мнимое или комплексное число в качестве нашего ответа, будет неопределенным в наборе действительных чисел. Мы не всегда очень четко говорим, с каким набором чисел мы работаем, но на протяжении этого занятия мы будем рассматривать только действительные числа (обратите внимание, что на наших графиках нет возможности отобразить воображаемое или сложное число). количество).

Например, вы должны были столкнуться с подобными задачами на предыдущем уроке алгебры


  1. 2/0 не определено, потому что у нас нет хорошего способа определить это математически, не приведя к противоречию. Например, предположим, что это значение определено и фактически равно некоторому числу, которое мы решили назвать n . Тогда по определению мы бы имели:

    Но это противоречие! Два НЕ равно нулю!
    На самом деле, мы замечаем, что НЕТ значения, которое мы могли бы подставить для n в приведенном выше уравнении, которое сделало бы это уравнение верным, потому что независимо от того, какое значение мы пытаемся использовать для n , утверждение 2 = n ·0 НИКОГДА не будет истинным. Таким образом, мы никак не можем понять число, имеющее ноль в знаменателе, потому что нет никакого способа определить единственное значение, равное этому числу.


  2. 0/0 не определено, потому что, как и 2/0, у нас нет хорошего способа определить это математически, не приведя к противоречию. Например, предположим, что это значение определено и фактически равно некоторому числу, которое мы решили назвать 9.0013 п . Тогда по определению у нас будет:

    Сначала это кажется правильным, потому что любое значение, которое мы подставим для n , сделает уравнение верным. Однако проблема именно в этом: ЛЮБОЕ значение, которое мы подставим вместо n , сделает уравнение верным, поэтому 0/0 можно определить как множество различных НЕРАВНЫХ возможных значений. Другими словами, ему нельзя присвоить только одно значение, не присвоив ему также другие неравные значения. Чтобы понять, почему это так, давайте рассмотрим простое уравнение:

    Но это противоречие! Один НЕ равен двум!
    Таким образом, мы никак не можем понять число, в знаменателе которого ноль, даже если оно также имеет ноль в числителе.


  3. В этом случае не определено в наборе действительных чисел всякий раз, когда n отрицательно, потому что в этом случае будет получено мнимое число. Поскольку не может быть равно никакому РЕАЛЬНОМУ числу, когда n отрицательно, оно не определено в наборе действительных чисел (но не в наборе комплексных чисел, который включает мнимые числа).


  4. Это пример неопределенного выражения, которое вы, возможно, раньше не видели. Однако мы можем быстро увидеть, что оно не определено, потому что его можно переписать как выражение 0/0, которое, как мы уже знаем, является неопределенным:

    Поскольку 0/0 не определено, 0 0 также должно быть неопределенным, поскольку мы имеем только что показал, что эти два выражения эквивалентны.
    (Вообще-то иногда математики решают считать 0 0 равным 1, хотя неясно, так ли это — это скорее условность. Чтобы прочитать интересное обсуждение того, как и почему это делается, посмотрите на этой странице!)

Когда мы алгебраически вычисляем предельные задачи, мы часто получаем в качестве исходного ответа что-то неопределенное. Это связано с тем, что «интересными» местами для поиска пределов являются места, где функция undefined . Поскольку функция f(x) не определена при x=c , f(c) даст неопределенное выражение. Однако нам важно помнить, что при расчете предела f(x) как x→c нас интересует не поведение f(x) AT c , а скорее поведение f(x) ВОКРУГ c . Итак, это приводит нас к мотивирующему вопросу этой лекции:

.

Когда мы получаем неопределенное значение в f(c) , может ли тип полученного неопределенного значения рассказать нам что-нибудь о поведении f(x) ВОКРУГ x=c ?

В оставшейся части лекции мы поиграем с примерами предельных задач, пытаясь ответить на этот вопрос!

Начнем с того, что вспомним Пример №2 из прошлой лекции:

График f(x) представлял собой линию с дыркой в ​​точке x = -2:

В этом случае, когда мы заменили f(x) на x -2, мы на самом деле заменили f(x) , то есть линию с отверстием в точке x = -2, на y = x -2, то есть та же самая линия без отверстия. Эти две функции не полностью идентичны, но они идентичны везде, кроме точки 9.0013 x = -2, это все, что имеет значение при расчете лимита. Чтобы две функции имели один и тот же предел при x = -2, все, что нам нужно, это чтобы они были идентичны в некотором интервале вокруг x = -2 (но НЕ обязательно при x = -2).

Итак, подытожим шаги, с которыми мы столкнулись при решении этих задач:

  1. Мы попытались вычислить f(c) напрямую, но обнаружили, что оно не определено (в данном случае потому, что оно равно 0/0).

  2. Мы нашли способ заменить f(x) другой функцией, которая совпадает с f(x) везде, за исключением x=c (в этом случае путем факторизации верхней и нижней части дроби и сокращая общий множитель).

  3. Мы вычислили предел этой новой функции, заменив x на на , и на этот раз мы получили значение, которое не было неопределенным. Поскольку новая функция такая же, как и f(x) везде, кроме x=c , пределы обеих функций одинаковы, поэтому мы можем заключить, что предел f(x) одинаков.

Мы увидим, что та же самая закономерность встречается во многих предельных задачах, которые мы будем решать. Единственным основным отличием будет то, что иногда тип неопределенного значения, которое мы получаем, скажет нам что-то о поведении f(x) в интервале около x=c , а иногда неопределенная форма не даст нам достаточно информации о том, что происходит с f(x) вокруг x=c , и в этом случае нам нужно будет выполнить дополнительные шаги алгебры, как мы сделали выше, чтобы переписать f(x) так, чтобы подставить c для x даст нам конкретную информацию о поведении f(x) вокруг x=c .

Давайте рассмотрим несколько примеров.

Но перед тем, как мы углубимся в примеры, давайте немного проясним некоторые обозначения:

Обозначение: Использование 0 и ∞ при расчете пределов.

В прошлой лекции мы видели, как мы можем вычислить f(c) в качестве одного шага к попытке определить предел f(x) , когда x приближается к c . В случаях, когда f(c) существует, это просто, потому что тогда предел будет равен f(c) . Однако в большинстве случаев мы вычисляем предел именно потому, что f(c) НЕ существует, и в этих случаях вычисление f(c) всегда будет давать неопределенное выражение. В этих случаях, когда мы вычисляем f(c) , на самом деле мы думаем о том, что такое f(c + ) и f(c ) .

Другими словами, мы должны помнить, что , когда мы пытаемся оценить предел, подставляя c вместо x , мы НЕ подставляем значение c точно , а скорее мы подстановка значений, произвольно БЛИЗКИХ к c , но фактически НЕ РАВНЫХ c.

Ноль:

Например, если мы говорим, что как x→c , на самом деле мы имеем в виду, что f(x) является дробью, для которой верхнее и нижнее значения произвольно близки к ноль по мере того, как x становится все ближе и ближе к c . Однако ни верх, ни низ дроби никогда не достигают нуля. Другими словами, и верх, и низ f(x) уменьшаются по величине по мере того, как x становится все ближе и ближе к с . Таким образом, нули в выражении INSERT на самом деле НЕ являются нулями — скорее, они заменяют числа, которые имеют очень маленькую величину (т. е. очень близки к нулю) .

Бесконечность:

Точно так же, когда мы используем обозначение ∞ при вычислении пределов, мы на самом деле не имеем в виду бесконечность. Помните, что то, что мы подразумеваем под ±∞, на самом деле является просто паттерном неограниченного поведения, когда величина чисел неограниченно возрастает.

Итак, например, если я нахожу, что x → -∞, то на самом деле это означает, что f(x) является выражением, для которого первый и второй члены оба становятся произвольно большими по величине как x становится все более и более отрицательным . Однако ни первый, ни второй член выражения никогда не достигают бесконечности, потому что это невозможно. Бесконечность — это не то число, которого можно достичь. Другими словами, и первый, и второй член в f(x) неограниченно растут по величине, поскольку x становится все более и более отрицательным . Итак, знаки бесконечности в выражении INSERT NOTATIONEX2.GIF HERE на самом деле НЕ являются бесконечностями — скорее, они заменяют числа, которые имеют очень большие величины (т. е. очень и очень далеки от нуля) .

При вычислении предельных выражений и 0, и ∞ заменяют тип ПОВЕДЕНИЯ ВОКРУГ

x=c :
  • 0 обозначает некоторое число, сколь угодно близкое к нулю;

  • +∞ обозначает произвольно большое число; и

  • -∞ обозначает некоторое число, которое является отрицательным, но имеет сколь угодно большую величину.

Теперь давайте перейдем к этим примерам!

Алгебраическое вычисление пределов: примеры

Пример 1: Когда

f(c) дает неопределенное выражение a/0, где a≠0

В этом примере, когда мы вычисляем f(c) , мы сначала получим выражение вида a/0, где a ≠0 (т. е. дробь, где верхнее число представляет собой некоторое фиксированное ненулевое значение, а нижнее число равно нулю):

В этом случае f(x) → -∞ как x → -2 справа, потому что f(x) приближается как x приближается к -2 справа. Другими словами, когда x приближается к -2 справа, числитель f(x) становится очень близким к -2, в то время как величина знаменателя становится все меньше. Если мы разделим числа, сколь угодно близкие к -2, на положительные числа со все меньшей величиной, в результате мы получим отрицательные числа со все большей величиной. И если поместить в знаменатель числа с достаточно малой величиной, мы можем получить числа с такой большой величиной, как мы хотим — так что это поведение неограниченно. В результате можно сказать, что f(x) будет неограниченно уменьшаться как x приближается к -2 справа, и мы можем написать, что f(x) → -∞ как x → -2 .

Пример 2: Когда

f(c) дает неопределенное выражение 0/0, но фактический предел f(x) , поскольку x приближается к c , является определенным конечным ненулевым числом

В этом примере, когда мы вычисляем f(c) , мы сначала получим выражение вида 0/0, но после замены f(x) 9 с помощью алгебры0014 с аналогичным выражением, которое совпадает с (но НЕ AT) x=c , мы сможем рассчитать фактический предел. Этот предел окажется конкретным конечным числом, не равным нулю в данном случае.

В этом случае f(x) приближается к 0/0, когда x приближается к 0. Другими словами, когда x приближается к 0, величины как числителя, так и знаменателя f(x) растут. все меньше. Этой информации недостаточно, чтобы сделать какой-либо вывод о пределе, потому что деление чисел со все более малой величиной на другие числа со все более меньшей величиной может привести к ряду различных результатов: зависит от того, насколько «маленькая» величина числителя по сравнению со знаменателем ! И мы еще ничего не знаем об отношении между числителем и знаменателем.

Итак, в этом случае нам нужно искать способ заменить f(x) аналогичной функцией, которая будет такой же, как f(x) все ВОКРУГ x=c , но не обязательно AT х=с . Для этого мы спросим себя, есть ли какая-либо алгебра, которую мы могли бы использовать, чтобы переписать f(x) без изменения его значения нигде, кроме x=c . В этом случае, поскольку f(x) содержит радикал в числителе, один из возможных подходов для нас состоит в том, чтобы попытаться переписать выражение так, чтобы радикал в числителе был сокращен — это может затем позволить нам что-то отменить. верха и низа дроби. (Мы не сможем полностью избавиться от радикала и сохранить функцию примерно x=c , но мы можем переместить ее, например, из числителя в знаменатель.)

Прежде чем перейти к другим примерам, которые помогут нам лучше понять, что может произойти, когда мы получим неопределенную форму 0/0 для f(c) , давайте на минутку отметим, как мы подошли к решению этой проблемы, которая будет один и тот же базовый подход для ВСЕХ примеров в этой лекции (и вообще для алгебраического вычисления пределов):

Большая идея: алгебра — это всего лишь способ поиска структуры.

Нам часто нужно, чтобы выражения, уравнения или другие математические объекты имели КОНКРЕТНУЮ СТРУКТУРУ, чтобы мы могли применить к ним определенное правило или использовать определенную технику.

Например, вы можете вспомнить, что на предыдущем уроке алгебры, когда вы хотели решить квадратное уравнение, вам нужно было, чтобы оно было в форме a x 2 + bx + c = 0, поэтому что вы могли бы разложить выражение в левой части уравнения, а затем установить каждый множитель равным нулю (потому что, если несколько вещей умножаются вместе, чтобы получить ноль, вы можете сделать вывод, что по крайней мере один из этих множителей должен быть равен нулю). Если вы столкнулись с квадратным уравнением, которое не было в этой форме (например, 4 — x 2 = -4x, например), вам нужно будет выполнить алгебраические операции над уравнением, чтобы вы могли заменить исходное уравнение эквивалентным уравнением, которое имеет форму, которую вы хотите . В этом случае два уравнения эквивалентны , если они имеют один и тот же набор решений (те же значения x , которые делают уравнение верным). Так, например, если бы я хотел поставить 4 — x 2 = -4x в виде a x 2 + bx + c = 0, я могу переставить члены уравнения так, чтобы оно выглядело так: 1 x 2 + -4x + -4 = 0. те же решения, что и исходное уравнение 4 — x 2 = -4x, но оно записано в нужной нам форме (потому что в этой новой форме легко разложить на множители, а затем решить).

Прямо сейчас мы заинтересованы в поиске пределов, и единственный способ, которым мы знаем, как найти пределы, — это просто подставить c для x и вычислить f(c) . Но иногда это не работает — иногда простое подключение c для x дает нам что-то, что не определено, например . Поэтому в этих случаях мы хотим спросить себя: «Какая основная структура в этом выражении приводит к тому, что оно становится неопределенным, когда я подставляю c вместо x , и есть ли способ заменить его другим? выражение, одинаковое везде вокруг x=c , но который не даст неопределенного результата, когда мы подключим c для x ?».

Таким образом, в будущем всякий раз, когда мы получим неопределённую форму для f(c) , первое, что мы спросим себя, это: «Как мы можем переписать f(x) , чтобы получить что-то эквивалентное (по крайней мере, вокруг x=c ), но который имеет другую структуру , которая поможет нам избежать именно этой неопределенной формы?

Пример 3: Когда

f(c) дает неопределенное выражение 0/0, но фактический предел f(x) при приближении x к c равен нулю

В этом примере, когда мы вычисляем f(c) , мы сначала получим выражение вида 0/0, но после замены f(x) с помощью алгебры на аналогичное выражение, которое совпадает с ( но НЕ В) x=c , мы сможем рассчитать фактический предел. В этом случае этот предел окажется равным нулю.

Как и в последнем примере, f(x) приближается к 0/0, когда x приближается к 0. Опять же, когда x приближается к 0, величины как числителя, так и знаменателя f(x) становятся все меньше, и этой информации недостаточно для того, чтобы сделать вывод о пределе, потому что зависит от того, насколько «маленькой» является величина числителя по сравнению со знаменателем , и, следовательно, зависит от соотношения между числитель и знаменатель .

Так же, как и в последнем примере, в этом случае нам нужно искать способ заменить f(x) аналогичной функцией, которая будет такой же, как f(x) все ВОКРУГ x=c , но не обязательно AT x=c . Снова мы спрашиваем себя, есть ли какая-нибудь алгебраическая формула, которую мы могли бы использовать, чтобы переписать f(x) , не изменяя его значение нигде, кроме x=c . В этом случае, поскольку f(x) содержит радикал в знаменателе, один из возможных подходов для нас состоит в том, чтобы попытаться переписать выражение так, чтобы радикал в знаменателе был сокращен — это может позволить нам что-то отменить. верха и низа дроби. (Опять же, как и в последнем примере, мы не сможем полностью избавиться от радикала и при этом сохранить ту же функцию около x=c , но мы можем переместить его, например, из знаменателя в числитель.)

Пример 4: Когда

f(c) дает неопределенное выражение 0/0, но f(x) → ± ∞, поскольку x приближается к c

В этом примере, когда мы вычисляем f(c) , мы сначала получим выражение вида 0/0, но после замены f(x) с помощью алгебры на аналогичное выражение, которое совпадает с ( но НЕ В) x=c , мы сможем рассчитать фактический лимит. В этом случае предела не будет, потому что f(x) будет неограниченно уменьшаться, когда x приближается к 0 слева, и неограниченно увеличиваться, когда x приближается к 0 справа.

Как и в последних двух примерах, f(x) приближается к 0/0, когда x приближается к 0. Опять же, когда x приближается к 0, величины как числителя, так и знаменателя f(x) становятся все меньше, и этой информации недостаточно для того, чтобы сделать какой-либо вывод о пределе, потому что зависит от того, насколько «маленькой» является величина числителя по сравнению со знаменателем , и, следовательно, зависит от отношения между числителем и знаменателем .

Так же, как и в последних двух примерах, в этом случае нам нужно искать способ заменить f(x) аналогичной функцией, которая будет такой же, как f(x) все ВОКРУГ x=c , но не обязательно AT x=c . Снова мы спрашиваем себя, есть ли какая-нибудь алгебраическая формула, которую мы могли бы использовать, чтобы переписать f(x) , не изменяя его значение нигде, кроме x=c . В этом случае, поскольку f(x) содержит радикал в числителе, один из возможных подходов для нас состоит в том, чтобы попытаться переписать выражение так, чтобы радикал в числителе был сокращен — это может затем позволить нам что-то отменить. верха и низа дроби. (Опять же, как и в последнем примере, мы не сможем полностью избавиться от радикала и при этом сохранить ту же функцию около x=c , но мы можем переместить его, например, из числителя в знаменатель.)

Эта проблема аналогична примеру 1 выше. В этом случае f(x) → -∞, поскольку x → 0 слева, потому что f(x) приближается, когда x приближается к 0 слева. Другими словами, когда x приближается к 0 слева, числитель f(x) становится очень близким к 1, в то время как величина знаменателя становится все меньше, и деление относительно фиксированного положительного числа (например, 1) на отрицательные числа со все меньшей величиной в результате мы получаем отрицательные числа со все большей величиной. И, как и в примере 1, это поведение не ограничено (поскольку, сделав величину знаменателя достаточно малой, мы можем получить числа с любой величиной, какой захотим). Таким образом, мы можем сказать, что f(x) будет неограниченно уменьшаться по мере приближения x к 0 слева, и мы можем написать, что f(x) → -∞ как x → 0 . Аналогично, f(x) →∞, поскольку x → 0 справа, потому что f(x) приближается к x , приближаясь к 0 справа.

Чем отличаются примеры 1, 2, 3 и 4?

Давайте вспомним эти четыре примера и суммируем различия между этими четырьмя похожими задачами. В каждом из этих примеров f(x) была дробью, которая имела ноль в знаменателе , когда мы заменили c на x , но в каждом случае числитель и знаменатель f(x) имели разные соотношения как x все ближе и ближе к c :

  1. В примере 1 по мере того, как x приближались к c , числитель f(x) приближался к фиксированному числу, а величина знаменателя уменьшалась на неопределенный срок. Это привело к числам с величиной, которая неограниченно увеличивалась (поскольку деление относительно фиксированного значения на числа, которые когда-либо приближаются к нулю, приводит к числам со все более высокой величиной, и поскольку, делая величину знаменателя достаточно малой, мы можем получить числа с такой большой величиной, как мы хотим).

  2. В примерах 2, 3 и 4 по мере того, как x приближались к c , значения как числителя, так и знаменателя f(x) уменьшались на неопределенный срок. Однако:

    1. В примере 2 величины числителя и знаменателя уменьшались примерно с «одной и той же» скоростью, так что при делении числителя на знаменатель мы получаем фиксированное значение, очень близкое к 1/2. (В этом случае мы не можем увидеть, что они будут «сжиматься примерно с одинаковой скоростью» напрямую; мы можем определить это, только сначала переписав функцию с помощью алгебры.)

    2. В примере 3 величина числителя уменьшалась гораздо быстрее, чем величина знаменателя, так что при делении числителя на знаменатель мы получаем значения, все более близкие к нулю. (В этом случае может быть трудно увидеть, что величина числителя будет уменьшаться «намного быстрее», но опять же, мы можем определить, что это так, переписав функцию с помощью алгебры. )

    3. В примере 4 величина знаменателя уменьшалась гораздо быстрее, чем величина числителя, так что при делении числителя на знаменатель мы получаем значения, имеющие все большую неограниченную величину. (В этом случае может быть трудно увидеть, что величина знаменателя будет уменьшаться «намного быстрее», но опять же, мы можем определить, что это так, переписав функцию с помощью алгебры.)

Итак, какой здесь более крупный паттерн?

Четыре примера, которые мы только что рассмотрели, показали нам, что:

  • Когда f(c) = a/0 для некоторого a ≠0, то этой информации достаточно, чтобы сказать нам, что f(x) → ± ∞ as x c , потому что деление относительно фиксированного значения на числа, которые все ближе к нулю, приводит к числам со все большей величиной, и потому, что, сделав величину знаменателя достаточно малой, мы можем получить числа с такой большой величиной, как мы хотим.

  • Когда f(c) = 0/0, то этой информации НЕ достаточно, чтобы сказать нам что-либо о том, что происходит с f(x) , поскольку x c , потому что это не говорит нам что-нибудь о соотношении между числителем и знаменателем . Мы знаем, что величины как числителя, так и знаменателя бесконечно уменьшаются, но мы не знаем, уменьшаются ли они примерно с одинаковой скоростью (и, следовательно, отношение числителя к знаменателю остается относительно постоянным) или если величина одного из них уменьшается «намного быстрее», чем другого (и, следовательно, отношение числителя к знаменателю либо уменьшается до нуля, либо неограниченно увеличивается/уменьшается).

    В этом случае мы должны использовать алгебру, чтобы заменить f(x) аналогичной функцией (то же самое, что f(x) ВОКРУГ, но не обязательно В x=c ) которая НЕ дайте нам 0/0, когда мы подключим c для x .

Заметим, что ОБА a/0 (когда a ≠0) и 0/0 не определены , но a/0 говорит нам кое-что о поведении предела (даже если оно не определено) , а 0/0 не дает нам никакой полезной информации о поведении лимита .

Итак, всякий раз, когда мы получаем неопределенное значение для f(c) , нам нужно будет остановиться и спросить себя, говорит ли полученная нами неопределенная форма что-нибудь о предельном поведении f(x) ПРИБЛИЗИТЕЛЬНО x= c или не . Это приводит нас к нескольким определениям, которые мы будем использовать для описания этого различия:

Определение: Неопределенные и определенные формы

Когда мы ищем lim x →c f(x) и подставляем c вместо x , получается неопределенное выражение для f(c) равно определить , если это дает нам достаточно информации, чтобы определить, каково предельное поведение f(x) ВОКРУГ x=c , без необходимости выполнять дальнейшие вычисления. (например, a/0 для a ≠0)

Это неопределенное выражение будет неопределенным , если существует более одного возможного типа предельного поведения f(x) ПРИБЛИЗИТЕЛЬНО x=c , которое может привести к этому конкретному неопределенному выражению. Другими словами, неопределенная форма не дает нам достаточно информации, чтобы определить, как ведет себя f(x) ВОКРУГ x=c , поэтому нам придется выполнить дальнейшие вычисления, чтобы понять это. (например, 0/0)

Осторожно! Обратите внимание, что эти определения имеют значение только при расчете лимита. Если я просто решаю задачу по алгебре и получаю в качестве ответа a/0 или 0/0, мой окончательный ответ на эту задачу будет просто таким: undefined . Мне было бы неправильно в этом контексте говорить что-либо об определенных или неопределенных формах, потому что я не исчисляю предела!

Теперь давайте вернемся к еще нескольким примерам, которые дают нам другие неопределенные выражения, когда мы вычисляем f(c) , и давайте посмотрим, сможем ли мы определить, какие неопределенные значения для f(c) являются неопределенными, а какие определенными формами!

Пример 5: Когда

f(c) дает неопределенное выражение b /±∞

В этом примере, когда мы вычисляем f(c) , мы сначала получим выражение вида b/±∞ (т. е. дробь, где верхнее число — некоторое фиксированное значение, а нижнее — бесконечность):

В данном случае f(x) → 0 как x → -∞, потому что f(x) приближается по мере неограниченного роста величины x . Другими словами, поскольку x безгранично уменьшается (то есть становится все более и более отрицательным), числитель f(x) становится очень близким к 3, а величина знаменателя становится все больше. Если мы разделим числа, сколь угодно близкие к 3, на отрицательные числа со все большей величиной, в результате мы получим отрицательные числа со все более меньшей величиной. Так мы получаем числа, которые все ближе и ближе к нулю. В результате можем написать, что f(x) → 0 как x → -∞ .

Пример 6: Когда

f(c) дает неопределенное выражение ±∞/ b

В этом примере, когда мы вычисляем f(c) , мы сначала получим выражение вида ±∞/b (т. е. дробь, где нижнее число — некоторое фиксированное значение, а верхнее — бесконечность):

В этом случае f(x) → +∞, поскольку x → +∞, потому что f(x) приближается к величине x растет без ограничений. Другими словами, по мере неограниченного увеличения x знаменатель f(x) становится очень близким к 0, а величина числителя становится все больше. Если мы разделим положительные числа, имеющие все большую величину, на положительные числа со все меньшей величиной (т. е. близкие к нулю), в результате мы получим положительные числа со все большей величиной. В результате мы можем написать, что f(x) → +∞ при х → +∞ .

Пример 7: Когда

f(c) дает неопределенное выражение ±∞/±∞, но фактический предел f(x) , поскольку x приближается к c , равен нулю

В этом примере, когда мы вычисляем f(c) , мы сначала получим выражение вида ±∞/±∞, но после замены f(x) с помощью алгебры на аналогичное выражение, которое является тем же как x → -∞, мы сможем рассчитать фактический предел. В этом случае этот предел окажется равным нулю.

В этом случае f(x) приближается к ∞/-∞, как x приближается к -∞. Другими словами, по мере неограниченного роста величины x величины как числителя, так и знаменателя f(x) становятся все больше. Этой информации недостаточно, чтобы сделать какой-либо вывод о пределе, потому что деление чисел со все большей величиной на другие числа со все большей величиной может дать ряд различных результатов: это зависит от того, насколько «велика» величина числителя по сравнению со знаменателем ! И мы еще ничего не знаем об отношении между числителем и знаменателем.

Итак, в этом случае нам нужно искать способ заменить f(x) на аналогичную функцию, которая будет такой же, как f(x) как x → -∞ (но не обязательно везде в других местах). ) . Для этого мы спросим себя, есть ли какая-либо алгебра, которую мы могли бы использовать, чтобы переписать f(x) без изменения его значения на отрицательные x -значения с особенно большой величиной. В этом случае проблема, по сути, состоит в том, что в есть x , как в числителе, так и в знаменателе: это означает, что всякий раз, когда мы подставляем -∞ вместо x , мы неизбежно получим знак бесконечности. и числитель и знаменатель. Итак, нам нужно придумать что-то, что мы можем сделать, чтобы «переписать» f(x) так, чтобы мы могли избавиться от x либо в числителе, либо в знаменателе. Это изменило бы форму, которую мы получаем, когда мы вычисляем f(c) из неопределенной формы ±∞/±∞ в определенную форму, которая равна либо b/±∞, либо ±∞/b, и мы знаем, что и что .

Итак, чтобы сделать это, мы начнем с того, что заметим, что наибольшая степень x в числителе равна x 2 : Итак, если бы мы разделили все в числителе и знаменателе на x 2 , мы могли бы «сократить» степени x в числителе, чтобы числитель не стремился к бесконечности, когда мы подставляем -∞ для x :

Существует несколько способов решения этой проблемы. Метод, использованный выше, является лишь одним из примеров, но предел также можно найти другим способом, используя аналогичный алгебраический метод, но на этот раз путем деления на наибольшую степень x в целом, а не просто на наибольшую степень 9.0013 х в числителе. Обратите внимание, что оба метода работают одинаково хорошо, помогая нам найти предел, давая одинаковые ответы в обоих случаях:

.

Пример 8: Когда

f(c) дает неопределенное выражение ±∞/±∞, но фактический предел f(x) при приближении x к c является фиксированным ненулевым числом

В этом примере, когда мы вычисляем f(c) , так же, как и в предыдущем примере, мы сначала получим выражение вида ±∞/±∞, но на этот раз после замены алгеброй f(x) с аналогичным выражением, которое совпадает с x → -∞, мы сможем вычислить фактический предел. Этот предел окажется равным 2.

Аналогично последнему примеру, f(x) приближается к ∞/∞, как x приближается к -∞. Как и прежде, по мере неограниченного роста величины x , величины как числителя, так и знаменателя f(x) растут все больше, и опять-таки этой информации недостаточно, чтобы сделать какой-либо вывод о ограничение, потому что мы ничего не знаем о соотношение между числителем и знаменателем еще.

Итак, как и в предыдущей задаче, нам нужно искать способ заменить f(x) аналогичной функцией, которая будет такой же, как f(x) при x → -∞, и снова мы замечаем, что наибольшая степень x в числителе равна x 2 : Итак, как и в последнем примере, если бы мы разделили все в числителе и знаменателе на x 2 , мы могли бы «обнулить» степени x в числителе, чтобы числитель не стремился к бесконечности, когда мы подставляем -∞ для x :

Пример 9: Когда

f(c) дает неопределенное выражение ±∞/±∞, но f(x) → ± ∞, поскольку x приближается к c

В этом примере, когда мы вычисляем f(c) , так же, как и в последних двух примерах, мы сначала получим выражение вида ±∞/±∞, но на этот раз после замены алгеброй f(x) с аналогичным выражением, которое совпадает с x → -∞, мы найдем, что f(x) →-∞ как x → -∞.

Аналогично последнему примеру, f(x) приближается к -∞/∞, как x приближается к -∞. Как и прежде, по мере неограниченного роста величины x , величины как числителя, так и знаменателя f(x) растут все больше, и опять-таки этой информации недостаточно, чтобы сделать какой-либо вывод о ограничение, потому что мы ничего не знаем о соотношение между числителем и знаменателем еще.

Так же, как и в последних двух задачах, нам нужно искать способ заменить f(x) аналогичной функцией, которая будет такой же, как f(x) при x → -∞, и снова используя тот же подход, что и в этих задачах, мы замечаем, что наибольшая степень х в числителе равна в числителе и знаменателе на x 3 , мы могли бы «обнулить» степени x в числителе, чтобы числитель не стремился к (отрицательной) бесконечности, когда мы подставляем -∞ для x :

Какой рисунок больше в данном случае?

Пять примеров, которые мы только что рассмотрели, показали нам, что:

  • Когда f(c) = b/±∞, то этой информации достаточно, чтобы сказать нам, что f(x) → 0 как x c , потому что деление относительно фиксированного значения на числа с возрастающей величиной приводит к числам, которые все ближе к нулю.

  • Когда f(c) = ±∞/b, то этой информации достаточно, чтобы сказать нам, что f(x) → ±∞ как x c , поскольку деление значения на — увеличение величины со значением, которое остается относительно фиксированным, приводит к числам со все большей и большей величиной, и мы можем получить в результате число любой величины, сделав величину числителя достаточно большой.

  • Когда f(c) = ±∞/±∞, то этой информации НЕ достаточно, чтобы сказать нам что-нибудь о том, что происходит с f(x) как x c , потому что это не Расскажите нам что-нибудь об отношении между числителем и знаменателем . Мы знаем, что величины как числителя, так и знаменателя неограниченно растут, но мы не знаем, растут ли они примерно с одинаковой скоростью (и, следовательно, отношение числителя к знаменателю остается относительно постоянным) или если величина одного из них растет «гораздо быстрее», чем другого (и, следовательно, отношение числителя к знаменателю либо уменьшается до нуля, либо неограниченно увеличивается/убывает).

    В этом случае мы должны использовать алгебру, чтобы заменить f(x) аналогичной функцией (то же самое, что f(x) ВОКРУГ, но не обязательно В x=c ) которая НЕ дайте нам ±∞/±∞, когда мы подставим c вместо x .

До сих пор мы рассматривали две категории детерминантных и недетерминантных форм:

  1. a/0, где a ≠0, — это определенная форма, стремящаяся к ±∞, а 0/0 — неопределенная форма.

  2. b/±∞ — детерминированная форма, стремящаяся к 0; ±∞/b — детерминантная форма, стремящаяся к ±∞; и ±∞/±∞ — неопределенная форма.

Но это не единственные два примера форматов, производящих детерминантные и недетерминантные формы. Есть ряд других детерминантных и недетерминированных форм, с которыми мы столкнемся, пытаясь решить предельные задачи алгебраически. Вот таблица, которая показывает все детерминантные и недетерминантные формы:

Справочная таблица: неопределенные и детерминированные формы

Определяющие и неопределенные формы
Здесь ∞ = +∞, a, b, и c — произвольные фиксированные действительные числа, a ≠0, c ≠1, c >0.
Неопределенные формы Определяющие формы

Осторожно! Заметим, что когда выражение содержит символ ± более чем в одном месте, ± не обязательно означает одно и то же в обоих местах! Например, если у нас есть a ·±∞ → ±∞, знак ± слева от стрелки и знак ± справа от стрелки могут не иметь одного и того же знака: если a отрицательно, они будут иметь противоположные знаки, например .

Таким образом, каждый раз, когда мы вычисляем f(c) , подставляя c вместо x , когда наша цель действительно состоит в том, чтобы найти предел f(x), поскольку x приближается к c, мы знаем, что если результат находится в списке неопределенных форм выше, нам нужно будет проделать дополнительную работу, прежде чем мы сможем вычислить предел (обычно путем перестановки f(x) с помощью некоторой алгебры). Однако, если выражение, которое мы получаем для f(c) , находится в списке форм определителя, мы уже знаем, каким будет предел f(x) , когда x приближается к c .

Но мы не хотим просто слепо использовать этот список! Если мы просто будем искать значения в этом списке, не понимая, почему выражения слева неопределимы, а выражения справа являются определяющими, мы, вероятно, в какой-то момент совершим ошибку и применим эти идеи неправильно. Кроме того, нам гораздо легче понять, почему каждая из этих форм является определяющей или неопределенной, чем просто запомнить список, не понимая его. Легко забыть список выражений, которые мы запомнили, но гораздо труднее забыть идею, которую мы на самом деле понимаем. Итак, Настоятельно рекомендую убедиться, что вы понимаете, как объяснить своими словами, почему каждая из этих форм является либо неопределенной, либо определяющей (и если она является определяющей, то каково будет значение предела).

Мы уже рассмотрели примеры и обсудили, как мы классифицировали первые две строки таблицы как определяющие или неопределенные, поэтому теперь давайте рассмотрим некоторые другие выражения:

В чем разница между ∞ — ∞ и ∞+∞?

В третьей строке нашей таблицы мы замечаем, что ∞ — ∞ (или -∞+∞) является неопределенным, а ∞+∞ (или -∞ — ∞) является определяющим — почему это так? Давайте подумаем об этом, а затем разработаем несколько предельных примеров. Мы видим, что ∞+∞ должно стремиться к ∞, потому что сложение двух значений вместе, оба из которых неограниченно возрастают, просто даст нам третье значение, которое также неограниченно возрастает. (Аналогично -∞ — ∞ даст нам что-то, что неограниченно убывает.)

Однако, если мы подумаем об ∞ — ∞, мы увидим, что мы сталкиваемся с проблемой, заключающейся в том, что мы не знаем отношения между первым и вторым значением:

  • Возможно, величина первого значения увеличивается «гораздо быстрее», чем второе значение, и в этом случае ∞ — ∞ будет стремиться к +∞.

  • Возможно, величина второго значения увеличивается «должно быстрее», чем первое значение, и в этом случае ∞ — ∞ будет стремиться к -∞.

  • Или может случиться так, что величины как первого, так и второго значения увеличиваются «примерно с одинаковой» скоростью, и в этом случае ∞ — ∞ будет стремиться к 0 или какому-то другому фиксированному значению.

Итак, пока мы не узнаем больше о соотношении между первым и вторым значением в выражении ∞ — ∞, мы не знаем, что делать с предельным поведением f(x) около x=c.

Это также легко представить графически: мы можем думать о выражении ∞ — ∞ как описывающем два графика (один график для первого члена и один для второго члена), каждый из которых неограниченно возрастает, а затем ∞ — ∞ означает расстояние между двумя графиками по мере приближения x c. Если первый и второй график представляют собой две параллельные линии с положительным наклоном, каждая линия будет неограниченно расти как x →∞, но расстояние между двумя линиями останется фиксированным как x →∞. Однако, если одна из этих линий круче другой, расстояние между двумя линиями будет увеличиваться как x →∞.

Давайте посмотрим на некоторые проработанные примеры для этих различных случаев детерминантной формы ∞+∞ (или -∞ — ∞) и неопределенной формы ∞ — ∞:

Пример 10: Когда

f(c) дает неопределенное выражение ∞+∞, поэтому f(x) → ∞, поскольку x приближается к c

Пример 11: Когда

f(c) дает неопределенное выражение ∞ — ∞, но f(x) → ∞, поскольку x приближается к c

Заметим, что в этом случае величина первого члена растет «быстрее», чем величина второго члена.

Пример 12: Когда

f(c) дает неопределенное выражение ∞ — ∞, но f(x) → -∞, поскольку x приближается к c

Заметим, что в этом случае величина второго члена растет «быстрее», чем величина первого члена.

Пример 13: Когда

f(c) дает неопределенное выражение ∞ — ∞, но f(x) приближается к нулю, когда x приближается к c

Заметим, что в этом случае величины как первого, так и второго членов растут примерно с «одной и той же» скоростью.

Пример 14: Когда

f(c) дает неопределенное выражение ∞ — ∞, но f(x) приближается к фиксированному конечному ненулевому значению, когда x приближается к c

Заметим, что в этом случае величины как первого, так и второго членов растут примерно с «одной и той же» скоростью.

Теперь, когда мы изучили детерминантную форму ∞+∞ (или -∞ — ∞) и недетерминированную форму ∞ — ∞, давайте посмотрим на различные случаи детерминантных форм a ·±∞ и ±∞·±∞, а неопределенная форма 0·±∞:

В чем разница между 0·±∞ и двумя случаями

и ·±∞ и ±∞·±∞?

В четвертой строке нашей таблицы мы замечаем, что 0·±∞ неопределенно, а a ·±∞ и ±∞·±∞ являются определяющими – почему это так? Мы можем видеть, что a ·±∞ и ±∞·±∞ должны стремиться к ±∞, потому что умножение двух значений вместе, оба из которых имеют неограниченно возрастающие величины, просто даст нам третье значение, величина которого также возрастает. без ограничений (хотя его знак будет зависеть от знаков перемножения двух множителей).

Однако, если мы подумаем о 0·±∞, то увидим, что мы сталкиваемся с проблемой, состоящей в том, что мы не знаем отношения между первым и вторым факторами:

  • Возможно, величина первого фактора уменьшается «гораздо быстрее», чем величина второго фактора увеличивается, и в этом случае 0·±∞ будет стремиться к 0. Например, подумайте о следующих последовательностях значений, и подумайте, что произойдет, если мы умножим каждый из их членов вместе:

    Умножение каждого члена из первой последовательности на каждый член из второй последовательности дает нам:

  • Возможно, величина второго фактора увеличивается «должно быстрее», чем уменьшается величина первого фактора, и в этом случае 0·±∞ будет стремиться к ±∞. Например, подумайте о следующих последовательностях значений и подумайте, что произойдет, если мы умножим каждый из их членов вместе:

    Умножение каждого члена из первой последовательности на каждый член из второй последовательности дает нам:

  • Или может случиться так, что величина первого фактора уменьшается «примерно с той же» скоростью, что и величина второго фактора увеличивается, и в этом случае 0·±∞ будет стремиться к какому-то другому фиксированному значению. Например, подумайте о следующих последовательностях значений и подумайте, что происходит, когда мы умножаем каждый из их членов вместе:

    Умножение каждого члена из первой последовательности на каждый член из второй последовательности дает нам:

Таким образом, пока мы не узнаем больше о соотношении между первым и вторым значением в выражении 0·±∞, мы не знаем, что можно сказать о предельном поведении f(x) при x в.

Мы еще не обсуждали последние три строки таблицы, в которой перечислены неопределенные и определенные формы. Мы потратим несколько минут, чтобы обрисовать в общих чертах идеи, лежащие в основе каждой из этих форм, но мы оставим вам в качестве дополнительной награды для этого класса, чтобы вы привели конкретные примеры каждой из этих различных форм. Позднее в этом семестре мы столкнемся с некоторыми задачами на предельные значения, которые придадут этим определяющим и недетерминированным формам, но обычно нам потребуются более сложные инструменты для решения этих задач на предельные значения, а мы еще не знакомы с этими инструментами. (Однако, используя графики или метод проб и ошибок, вы можете найти примеры предельных задач, которые включают одну из этих трех последних неопределенных форм.)

The Indeterminate Forms 0

0 , 1 ±∞ , and ∞ 0 versus the Determinant Forms 0 ±∞ , a ±∞ , ∞ a , and ∞ ±∞

Давайте начнем с рассмотрения того, почему 0 0 является неопределенным, а 0 ±∞ является определяющим — почему это так? Мы можем видеть, что 0 ±∞ должно стремиться к 0, потому что умножение некоторого значения, величина которого постоянно уменьшается сама по себе во все большее и большее число раз, просто даст нам третье значение, величина которого также бесконечно уменьшается (т. е. стремится к 0).

Однако, если мы подумаем о 0 0 , мы увидим, что сталкиваемся с проблемой, заключающейся в том, что мы не знаем отношения между основанием и показателем степени:

  • Возможно, величина основания уменьшается «намного быстрее», чем величина показателя степени, и в этом случае 0 0 будет стремиться к 0. (Подсказка: подумайте о функции, в которой основание остается фиксированным на нуле, а показатель стремится к нулю).

  • Возможно, величина показателя степени убывает «гораздо быстрее», чем величина основания, и в этом случае 0 0 будет стремиться к 1. (Подсказка: подумайте о функции, у которой основание стремится к нулю, но показатель остается фиксированным на нуле).

Как и в других примерах, пока мы не узнаем больше об отношении между показателем степени и основанием в выражении 0 0 , мы не знаем, что можно сделать в отношении предельного поведения числа 9.0013 f(x) как x c.

Небольшое примечание к этому примеру для тех из вас, кто интересуется: 0 0 на самом деле не является неопределенным, потому что, если вы осмотритесь, вы можете найти некоторые доказательства того, что 0 0 = 0. Однако это не так. Это действительно имеет отношение к нашему изучению исчисления, потому что даже если 0 0 не является неопределенным, когда мы что-то точно вычисляем, когда мы находим предел f(x) , мы не получаем 0 0 точно; вместо этого мы пытаемся определить, как ведет себя f(x) по мере того, как оно стремится к 0 0 , что является еще одним способом задать вопрос, к какому значению приближается степень, когда и ее основание, и ее показатель степени стремятся к нулю (и мы мы не можем ответить на этот вопрос, если не знаем отношения между скоростью, с которой основание стремится к нулю, и скоростью, с которой показатель степени стремится к нулю).

Неопределенные формы 1
±∞ по сравнению с определяющей формой а ±∞

Теперь рассмотрим, почему 1 ±∞ неопределенно, а c ±∞ (для c ≠1 и c >0) является определяющим – почему это так?

Мы можем видеть, что когда c >1, c должно стремиться к ∞, потому что умножение некоторого положительного значения больше единицы само на себя все большее и большее число раз будет давать нам все большие и большие значения (и мы можно получить значение, которое мы хотим, просто сделав экспоненту настолько большой, насколько нам нужно для этого). Мы видим, что когда 0 < c <1, c должны стремиться к 0, потому что умножение некоторого положительного значения меньше единицы само на себя все большее и большее число раз будет давать нам значения со все меньшими и меньшими величинами (или значения, которые ближе и ближе ближе к нулю).

В связи с этим мы можем видеть, что когда c >1, c -∞ должно стремиться к 0, потому что c -∞ на самом деле просто 1/ c 2 ∞ 3 9, и мы уже знаю, что c →∞ (когда c >1) и 1/∞→0. Точно так же, когда 0 < c < 1, c -∞ должно стремиться к ∞, потому что c -∞ на самом деле просто 1/ c ∞ 3 c , и мы уже знаем, что → 0 (когда 0 < c < 1) и 1/0 → ∞ (когда знаменатель положителен, как здесь, потому что он приближается к 0 с положительной стороны).

Однако, если мы подумаем о 1 ±∞ , мы можем видеть, что мы сталкиваемся с проблемой, что мы не знаем отношения между основанием и показателем степени:

  • Возможно, величина основания стремится «гораздо быстрее» к 1, чем величина показателя степени стремится к бесконечности, и в этом случае 1 ±∞ будет стремиться к 1. (Подсказка: подумайте о функции, в которой основание остается фиксированным на единице, а показатель степени стремится к плюс или минус бесконечности).

  • Возможно, показатель степени стремится к положительной бесконечности «намного быстрее», чем основание стремится к 1, и что основание приближается к 1 с положительной стороны, так что значения в основании больше 1: в этом случае 1 будет стремиться к ∞.

  • Возможно, показатель степени стремится к положительной бесконечности «намного быстрее», чем основание стремится к 1, и что основание приближается к 1 с отрицательной стороны, так что значения в основании меньше 1: в этом случае 1 будет стремиться к 0,

  • Возможно, показатель степени стремится к отрицательной бесконечности «гораздо быстрее», чем основание стремится к 1, и что основание приближается к 1 с положительной стороны, так что значения в основании больше 1: в этом случае 1 -∞ будет стремиться к 0 (потому что 1 -∞ на самом деле просто 1/1 , а когда основание меньше 1, 1/1 → 1/∞ → 0).

  • Возможно, показатель степени стремится к отрицательной бесконечности «намного быстрее», чем основание стремится к 1, и что основание приближается к 1 с отрицательной стороны, так что значения в основании меньше 1: в этом случае 1 -∞ будет стремиться к ∞ (потому что 1 -∞ на самом деле просто 1/1 , а когда основание меньше единицы, 1/1 → 1/0 → ∞). (Мы знаем, что в этом случае 1/0 → ∞ вместо -∞, потому что 1 приближается к нулю с положительной стороны.)

Как и в других примерах, пока мы не узнаем больше о соотношении между показателем степени и основанием в выражении 1 ±∞ , мы не знаем, что можно сделать в отношении предельного поведения f(x) как x c.

Мы можем использовать аналогичные рассуждения, чтобы лучше понять неопределенные формы ∞
0 по сравнению с определяющими формами ∞ a и ∞ ±∞ , которые являются последним набором форм в нашей таблице.

Чтобы закончить эту лекцию, давайте рассмотрим еще несколько примеров, некоторые из которых используют приемы, которые мы не использовали в более ранних примерах задач.

Еще несколько примеров предельных задач, которые можно решить алгебраически:

Пример 15: Использование факторинга для устранения неопределенной формы 0/0

Для этого уравнения прямая подстановка c в f(x) снова даст нам 0/0, что не определено. Однако, в то время как а/0 не определено для всех значений а, доля, где верх остается фиксированным на ненулевом значении и где низ приближается (но не достигает) к нулю, фактически приближается к положительной или отрицательной бесконечности (в зависимости от знаков числителя и знаменателя). определить где f(x) может безгранично увеличиваться или уменьшаться (т. е. должна ли бесконечность иметь перед собой положительный или отрицательный знак), мы должны рассматривать каждый односторонний предел отдельно:

Найдите предел f(x) , когда x приближается к 0:

Пример 16: Использование факторинга для устранения неопределенной формы 0/0 с различиями в пределе, когда мы оцениваем его слева и справа

Эта функция аналогична последней функции; однако мы замечаем, что на этот раз правый и левый пределы различаются по знаку/направлению:

 

Пример 17: Использование деления на степень

x , даже если дробь включает знак корня, для устранения неопределенной формы ±∞/±∞

Эта функция аналогична примерам 7, 8 и 9, за исключением того, что здесь требуется модифицированный метод, чтобы переписать уравнение, чтобы его можно было вычислить путем подстановки. На этот раз из-за наличия радикала в числителе мы должны делить на квадратный корень из 9.0013 x 2 , а так как это значение всегда будет положительным, мы должны быть особенно осторожны, чтобы отслеживать знаки:

Нет никаких причин, по которым наш предел должен быть отрицательным, поскольку x становится «более отрицательным» (т. » (т.е. как x → + ∞). Например, у нас может быть противоположный случай, как в функции, представленной на следующем графике:

Пример 18: использование подстановки для оценки предела, который нельзя оценить с помощью одного из предыдущих методов

И, наконец, у нас есть функция, которая имеет колебательное поведение около x=c , и поэтому, чтобы вычислить предел здесь алгебраически, мы разбиваем задачу на два отдельных предельных вопроса:

 

зум

К этому моменту мы должны уметь находить все виды пределов, либо глядя на график функции, либо алгебраически манипулируя уравнением функции!

И мы также должны быть в состоянии объяснить, почему некоторые неопределенные значения, которые мы получаем, когда мы вычисляем f(c) , являются определяющими, а другие неопределенными

Стратегия поиска пределов — GeeksforGeeks

Пределы оказались действительно полезными в области исчисления, они стали прочной основой для определения многих понятий, таких как непрерывность, дифференцируемость, интегралы и производные. Эти концепции также помогают нам анализировать множество функций и их поведение в исчислении. Пределы были основой почти для всех концепций исчисления. Таким образом, становится необходимым научиться вычислять пределы для различных типов функций и как обращаться с неопределенными формами пределов. Давайте посмотрим на различные методы, которые помогают нам вычислять пределы для сложных функций и выражений.

Пределы

Рассмотрим функцию f(x) и точку x = c, предел в этой точке определяется как значение, которое функция, по-видимому, принимает при приближении к этому значению x = c либо слева- ручная или правая сторона. Предел функции в конкретной точке определяется как

Большинство пределов можно вычислить простой подстановкой точки x = a в функцию. Это называется методом прямой замены. Иногда при вычислении пределов мы можем столкнуться с некоторыми выражениями, которые не определены. Это неопределенные формы предела.

Например, рассмотрим функцию f(x) =. Цель состоит в том, чтобы найти предел этой функции при x = 2. 

Обратите внимание, что при прямой подстановке этот предел принимает вид 0/0. Это неопределенная форма, и она называется неопределенной формой. Точно так же ∞/∞, 1 также называются неопределенными формами. Для решения таких форм используется ряд стратегий.

Стратегии решения пределов

Существует несколько стратегий и методов, используемых для нахождения пределов функции. Какой метод будет использоваться для какой функции, зависит от нескольких факторов. Например, тип функции (тригонометрическая, экспоненциальная, полиномиальная и т. д.), встречающаяся неопределенная форма (∞/∞, 1 , 0/0 и т. д.). Для этих вещей нет установленных правил, нужно практиковаться, и это приходит с опытом, когда человек находит ограничения для различных видов функций. Давайте рассмотрим некоторые стратегии для преодоления ограничений.

Прямая замена 

Многие пределы можно оценить, просто подставив значение точки в функцию. Необходимым условием использования этого подхода является то, что функция должна быть непрерывной, а предел не должен давать на выходе какой-либо неопределенной формы.

Пример: рассмотрим функцию f (x) = x 2 + 4x + 13. Найти

Решение:

⇒1 +4 + 13

⇒1 +4 + 13

⇒1 +4 + 13

⇒1 +4 + 13

⇒ 18

Факторинг и отмена

Иногда в некоторых функциях при использовании метода подстановки предел принимает вид 0/0. Часто в этих случаях в числителе и знаменателе есть некоторые общие множители, которые можно разложить на множители и сократить.

Пример. Рассмотрим функцию f(x) =  . Найдите  

Решение:

Методом подстановки,

⇒ Методом факторинга.

Особый случай с функцией синуса

Иногда при оценке формы 0/0, если функция синуса присутствует. Это удостоверение пригодится.

Пример: Рассмотрим функцию f(x) = . Найти  

Решение:

Этот предел имеет вид 0/0.

с использованием идентификации, упомянутой выше,

Умножение на образу . Этот метод может быть использован для решения предела. В этом случае и числитель, и знаменатель делятся на наибольшую степень числа x, входящего в функцию.

Пример: Рассмотрим функцию f(x) = . Найдите  

Решение:

Этот предел имеет форму ∞/∞.

L’ -Possist. Правило

Это правило полезно для неопределенных форм, таких как 0/0 или ∞/∞. Нет ограничений на класс функций, к которым он может применяться. Его можно применять для любого типа функций, которые оцениваются в неопределенных формах с помощью метода подстановки. В этом правиле числитель и знаменатель дифференцируются до тех пор, пока предел не придет в детерминированную форму.

Пример: упомянутая выше функция f(x) =   . Найдите    , используя правило Лопиталя.

Решение: 

Дифференцирование числителя и знаменателя.

Теперь этот предел не в неопределенной форме,

Давайте рассмотрим еще несколько примеров этих методов.

Примеры задач

Вопрос 1. Рассмотрим функцию f(x) = x 3 + 4x 2 + 1. Найти

Решение:

⇒ ВОПРОСОВАЯ 2: ВОПРОСОВАЕТ 2: ВОПРОСОВАЕТ 2: ВОПРОСОВАЯ 2: ВОПРОСИЕ 2: ВОПРОСИЕ 2: ВОПРОСИЕ 2: ВОПРОСИЕ 2: 10003

. f(x) = . Найдите  

Решение:

Методом подстановки,

⇒ Методом факторинга.

⇒ -1

Вопрос 3: Рассмотрим функцию f(x) = . Найдите  

Решение:

Этот предел имеет форму ∞/∞.

⇒ 4

Найдите    , используя правило Лопиталя.

Решение:

Дифференцирование числителя и знаменателя.

Теперь этот предел не в неопределенной форме,

Вопрос 5: Рассмотрим функцию f(x) = . Найдите  

Решение:

Этот предел имеет форму ∞/∞.

Вопрос 6: Рассмотрим функцию f(x) =  . Найдите  

Решение:

Этот предел имеет форму ∞/∞.

Вопрос 7: Рассмотрим функцию F (x) = . Найдите  

Решение:

Этот предел имеет форму ∞/∞.

⇒ 2


Свойства ограничений · Precalculus

Нахождение пределов: свойства пределов · Предварительное исчисление

В этом разделе вы:

  • Найдите предел суммы, разности и произведения.
  • Найдите предел многочлена.
  • Найдите предел силы или корня.
  • Найдите предел частного.

Рассмотрим рациональную функцию

f(x)=x2−6x−7x−7

Функция может быть представлена ​​следующим образом:

f(x)=(x−7)(x+1)x−7, что дает нам f(x)=x+1,x≠7.

Означает ли это, что функция f 

совпадает с функцией  g(x)=x+1?

Нет. Функция f 

не имеет x=7 

в своем домене, но  g 

делает. Графически мы видим, что в графике  f(x)  9 есть дыра.0003

при х=7,

, как показано на [ссылке], и нет такой дыры в графике  g(x),

, как показано в [ссылка].

Итак, эти две разные функции также имеют разные пределы как  x 

приближается к 7?

Не обязательно. Помните, что при определении предела функции как  x 

подхода а,

имеет значение то, приближается ли результат к действительному числу, когда мы приближаемся к  x=a.

Существование предела не зависит от того, что происходит, когда x 

равно a.

Посмотрите еще раз на [ссылка] и [ссылка]. Обратите внимание, что на обоих графиках as x 

приближается к 7, выходные значения приближаются к 8. Это означает

limx→7f(x)=limx→7g(x).

Помните, что при определении предела важно, что происходит вблизи x=a,

не в x=a.

В этом разделе мы будем использовать различные методы, такие как переписывание функций путем факторизации, для оценки предела. Эти методы дадут нам формальное подтверждение того, что мы раньше совершали с помощью интуиции.

Нахождение предела суммы, разности и произведения

Построение графика функции или изучение таблицы значений для определения предела может быть громоздким и занимать много времени. Когда это возможно, более эффективно использовать свойства пределов , которые представляют собой набор теорем для нахождения пределов.

Знание свойств пределов позволяет нам напрямую вычислять пределы. Мы можем складывать, вычитать, умножать и делить пределы функций, как если бы мы выполняли операции над самими функциями, чтобы найти предел результата. Точно так же мы можем найти предел функции, возведенной в степень, возведя предел в эту степень. Мы также можем найти предел корня функции, взяв корень предела. Используя эти операции над пределами, мы можем найти пределы более сложных функций, найдя пределы их более простых составляющих функций.

свойства ограничений

Let A, K, A,

и B

представляют реальные числа, а F

и G

— это функции, такие, что LIMX → AF (x) =

и Limx → AG (х)=В.

Для пределов, которые существуют и являются конечными, свойства пределов приведены в [ссылка] limx→ak=k Постоянное время функции limx→a[k⋅f(x)]=klimx→af(x)=kA Сумма функций limx→a[f(x)+g(x)]=limx→af(x)+limx→ag(x)=A+B Различие функций limx→a[f(x)−g(x)]=limx→af(x)−limx→ag(x)=A−B Произведение функций limx→a[f(x)⋅g(x)]=limx→af(x)⋅limx→ag(x)=A⋅B Частное функций limx→af(x)g(x)=limx→af(x)limx→ag(x)=AB,B≠0 Функция возведена в степень limx→a[f(x)]n=[limx→∞f(x)]n=An, где n – натуральное число. n -й корень функции, где n — натуральное число limx→af(x)n=limx→a[f(x)]n=An Полиномиальная функция limx→ap(x)=p(a)

Алгебраическое вычисление предела функции

Вычислить limx→3(2x+5).

limx→3(2x+5)=limx→3(2x)+limx→3(5)Свойство суммы функций                     = 2limx→3(x)+limx→3(5) Постоянные умножения на свойство функции                     =2( 3)+5 Оценить                     =11

Оцените следующий предел: limx→−12(−2x+2).

26

Нахождение предела многочлена

Не все функции или их пределы связаны с простым сложением, вычитанием или умножением. Некоторые могут включать полиномы. Напомним, что многочлен — это выражение, состоящее из суммы двух или более слагаемых, каждое из которых состоит из константы и переменной, возведенных в неотрицательную целую степень. Чтобы найти предел полиномиальной функции, мы можем найти пределы отдельных членов функции, а затем сложить их вместе. Кроме того, предел полиномиальной функции as x 

подхода a 

эквивалентно простому вычислению функции для  a

.

Для заданной функции, содержащей многочлен, найти ее предел.

  1. Используйте свойства пределов, чтобы разбить многочлен на отдельные члены.
  2. Найдите пределы отдельных терминов.
  3. Сложите ограничения вместе.
  4. В качестве альтернативы оцените функцию для

    .

Алгебраическое вычисление предела функции

Вычислить limx→3(5×2).

limx→3(5×2)=5limx→3(x2)Константа, умноженная на свойство функции                 =5(32)Функция, возведенная в степень свойства                =45

Вычислить limx→4(x3−5).

59

Алгебраическое вычисление предела многочлена

Вычислить limx→5(2×3−3x+1).

limx→5(2×3−3x+1)=limx→5(2×3)−limx→5(3x)+limx→5(1)Сумма функций                              =2limx→5(x3)−3limx→5(x) +limx→5(1)Константа, умноженная на функцию                               =2(53)−3(5)+1 Функция, возведенная в степень                                =236Вычислить

Оцените следующий предел: limx→−1(x4−4×3+5).

10

Нахождение предела степени или корня

Когда предел включает степень или корень, нам нужно другое свойство, чтобы помочь нам оценить его. Квадрат предела функции равен пределу квадрата функции; то же самое относится и к высшим силам. Точно так же квадратный корень из предела функции равен пределу квадратного корня функции; то же верно и для высших корней.

Оценка предела мощности

Вычислить limx→2(3x+1)5.

Мы возьмем предел функции, когда  x 

приближается к 2, и возведем результат в степень 5 .

limx→2(3x+1)5=(limx→2(3x+1))5                    = (3(2)+1)5                       =75                      =16 807 900 900

Оцените следующий предел: limx→−4(10x+36)3.

−64

**Если мы не можем напрямую применить свойства предела, например, в limx→2(x2+6x+8x−2)

, можем ли мы определить предел функции, когда x

приближается к

?**

Да. Некоторые функции можно алгебраически переставить, чтобы можно было оценить предел упрощенной эквивалентной формы функции.

Нахождение предела частного

Нахождение предела функции, выраженной в виде частного, может оказаться более сложной задачей. Нам часто нужно переписать функцию алгебраически, прежде чем применять свойства предела. Если знаменатель равен 0, когда мы применяем свойства предела напрямую, мы должны переписать частное в другой форме. Один из подходов состоит в том, чтобы записать частное в факторизованной форме и упростить.

Учитывая предел функции в форме частного, используйте факторинг для его оценки.

  1. Полностью разложите числитель и знаменатель на множители.
  2. Упростите, разделив любые множители, общие для числителя и знаменателя.
  3. Оцените полученное ограничение, не забывая использовать правильный домен.

Оценка предела частного путем разложения на множители

Вычислить limx→2(x2−6x+8x−2).

Умножьте, где это возможно, и упростите.

limx→2(x2−6x+8x−2)=limx→2((x−2)(x−4)x−2) Разложите числитель на множители. =limx→2((x−2)(x−4)x−2) Отменить общие множители. =limx→2(x−4)Вычислить. =2−4=−2

Анализ

Когда предел рациональной функции не может быть оценен напрямую, факторизованные формы числителя и знаменателя могут упроститься до результата, который можно вычислить.

Обратите внимание, функция

f(x)=x2−6x+8x−2

эквивалентно функции

f(x)=x−4,x≠2.

Обратите внимание, что ограничение существует, даже если функция не определена при x = 2.

Оцените следующий предел: limx→7(x2−11x+287−x).

−3

Оценка предела частного с помощью ЖК-дисплея

Вычислить limx→5(1x−15x−5).

Найдите ЛП для знаменателей двух слагаемых в числителе и преобразуйте обе дроби так, чтобы ДП было их знаменателем.

Анализ

При определении предела рациональной функции, в которой члены добавлены или вычтены либо в числителе, либо в знаменателе, первым шагом является нахождение общего знаменателя добавляемых или вычитаемых членов; затем преобразуйте оба члена так, чтобы они имели этот знаменатель, или упростите рациональную функцию, умножив числитель и знаменатель на наименьший общий знаменатель. Затем проверьте, имеют ли результирующие числитель и знаменатель какие-либо общие делители.

Вычислить limx→−5(15+1×10+2x).

−150

Учитывая предел функции, содержащей корень, использовать сопряжение для оценки.

  1. Если приведенное частное не является неопределенным (00)  форма

    , оценить напрямую.

  2. В противном случае перепишите сумму (или разность) двух частных как одно частное, используя наименьший общий знаменатель (LCD) .
  3. Если в числителе есть корень, рационализируйте числитель; умножьте числитель и знаменатель на сопряжено с числителя. Напомним, что a±b 

    — конъюгаты.

  4. Упростить.
  5. Оцените полученный предел.

Вычисление предела, содержащего корень, с помощью сопряжения

Вычислить limx→0(25−x−5x).

limx→0(25−x−5x)=limx→0((25−x−5)x⋅(25−x+5)(25−x+5)) Умножить числитель и знаменатель на сопряженное. =limx→0((25−x)−25x(25−x+5)) Умножить: (25−x−5)⋅(25−x+5)=(25−x)−25. =limx→0(−xx(25−x+5)) Объедините похожие термины. =limx→0(−xx(25−x+5))Упростить −xx=−1. =−125−0+5Оценить. =−15+5=−110

Анализ

При определении предела функции с корнем в качестве одного из двух членов, где мы не можем вычислить напрямую, подумайте об умножении числителя и знаменателя на сопряжение членов.

Оцените следующий предел: limh→0(16−h−4h).

−18

Оценка предела частного функции с помощью факторизации

Вычислить limx→4(4−xx−2).

limx→4(4−xx−2) = limx→4((2+x)(2−x)x−2)Коэффициент. =limx→4((2+x)(2−x)−(2−x)) Вынесите −1 из знаменателя. Упрощать. =limx→4−(2+x)Вычислить. =−(2+4)                      =−4

Анализ

Умножение на сопряженное расширит числитель; вместо этого ищите факторы в числителе. Четыре — это полный квадрат, поэтому числитель имеет вид

a2−b2

и может быть разложен на множители как

(a+b)(a−b).

Оцените следующий предел: limx→3(x−3x−3).

23

Учитывая частное с абсолютными значениями, оцените его предел.

  1. Попробуйте разложить или найти ЖК-дисплей.
  2. Если предел не может быть найден, выберите несколько значений рядом и по обе стороны от входа, где функция не определена.
  3. Используйте числовые данные для оценки пределов с обеих сторон.

Оценка предела частного с абсолютными значениями

Вычислить limx→7\|x−7\|x−7.

Функция не определена при x=7,

, поэтому мы попробуем значения, близкие к 7 слева и справа.

Левый предел: \|6.9−7\|6,9−7=\|6,99−7\|6,99−7=\|6,999−7\|6,999−7=−1

Правый предел: \|7,1−7\|7,1−7= \|7.01−7\|7.01−7=\|7.001−7\|7.001−7=1

Поскольку левый и правый пределы не равны, предела нет.

Вычислить limx→6+6−x\|x−6\|.

−1

Получите доступ к следующему онлайн-ресурсу для получения дополнительных инструкций и практики со свойствами пределов.

  • Определение предела аналитически

Ключевые понятия

  • Свойства пределов можно использовать для выполнения операций над пределами функций, а не над самими функциями. См. [ссылка].
  • Предел полиномиальной функции можно найти, найдя сумму пределов отдельных членов. См. [ссылка] и [ссылка].
  • Предел функции, возведенной в степень, равен той же степени предела функции. Другой метод — прямая замена. См. [ссылка].
  • Предел корня функции равен соответствующему корню предела функции.
  • Один из способов найти предел функции, выраженной в виде частного, состоит в том, чтобы записать частное в факторизованной форме и упростить. См. [ссылка].
  • Другой способ нахождения предела сложной дроби — найти ЖК. См. [ссылка].
  • Предел, содержащий функцию, содержащую корень, может быть оценен с использованием сопряжения. См. [ссылка].
  • Пределы некоторых функций, выраженных в виде частных, можно найти с помощью факторизации. См. [ссылка].
  • Одним из способов оценки предела частного, содержащего абсолютные значения, является использование числовых свидетельств. Настройка его по частям также может быть полезной. См. [ссылка].

Упражнения по секциям

Устный

Приведите пример функции  f 

, предел которой при приближении  x 

к  a,

равен f(a).

Если  f 

является полиномиальной функцией, предел полиномиальной функции при приближении  x 

к  a 

всегда будет f(a).

Когда используется прямая подстановка для оценки предела рациональной функции, когда  x 

приближается к  a 

, и результат равен f(a)=00,

означает ли это, что предел  f 

не существует?

Что значит сказать, что предел  f(x),

при приближении  x 

к c,

не определен?

Это может означать либо (1) значения функции неограниченно увеличиваются или уменьшаются по мере того, как  x 

приближается к  c,

, либо (2) левый и правый пределы не равны.

Алгебраический

Для следующих упражнений оцените пределы алгебраически.

предел→0(3)

limx→2(−5xx2−1)

−103

limx→2(x2−5x+6x+2)

limx→3(x2−9x−3)

6

limx→−1(x2−2x−3x+1)

limx→32(6×2−17x+122x−3)

12

limx→−72(8×2+18x−352x+7)

limx→3(x2−9x−5x+6)

6

limx→−3(−7×4−21×3−12×4+108×2)

limx→3(x2+2x−3x−3)

не существует

limh→0((3+h)3−27h)

limh→0((2−h)3−8h)

−12

limh→0((h+3)2−9h)

limh→0(5−h−5h)

−510

limx→0(3−x−3x)

limx→9(x2−813−x)

−108

limx→1(x−x21−x)

limx→0(x1+2x−1)

1

limx→12(x2−142x−1)

limx→4(x3−64×2−16)

6

limx→2−(\|x−2\|x−2)

limx→2+(\|x−2\|x−2)

1

limx→2(\|x−2\|x−2)

limx→4−(\|x−4\|4−x)

1

limx→4+(\|x−4\|4−x)

limx→4(\|x−4\|4−x)

не существует

limx→2(−8+6x−x2x−2)

В следующем упражнении используйте данную информацию для оценки пределов: limx→cf(x)=3,

limx→cg(x)=5

limx→c [ 2f(x)+g(x) ]

6+5

limx→c [ 3f(x)+g(x) ]

limx→cf(x)g(x)

35

Для следующих упражнений оцените следующие пределы.

limx→2cos(πx)

limx→2sin(πx)

0

limx→2sin(πx)

f(x)={2×2+2x+1,x≤0x−3, x>0; limx→0+f(x)

−3

f(x)={2×2+2x+1,x≤0x−3, x>0; limx→0−f(x)

f(x)={2×2+2x+1,x≤0x−3, x>0; limx→0f(x)

не существует; правый предел не совпадает с левым пределом.

лимкс→4х+5-3х-4

limx→2+(2x−〚x〛)

2

limx→2x+7−3×2−x−2

limx→3+x2x2−9

Предел не существует; предел приближается к бесконечности.

Для следующих упражнений найдите среднюю скорость изменения f(x+h)−f(x)h.

ф(х)=х+1

f(x)=2×2−1

4x+2h

f(x)=x2+3x+4

f(x)=x2+4x−100

2x+h+4

f(x)=3×2+1

f(x)=cos(x)

cos(x+h)−cos(x)h

f(x)=2×3−4x

f(x)=1x

−1x(x+h)

f(x)=1×2

f(x)=x

−1x+h+x

Графический

Найдите уравнение, которое можно представить с помощью [ссылка].

Найдите уравнение, которое можно представить с помощью [ссылка].

f(x)=x2+5x+6x+3

Для следующих упражнений см. [ссылка].

Каков правый предел функции, когда  x 

приближается к 0?

Каков левый предел функции, когда  x 

приближается к 0?

не существует

Реальные приложения

Функция положения s(t)=−16t2+144t 

дает положение снаряда как функцию времени. Найти среднюю скорость (среднюю скорость изменения) на интервале [1,2]

.

Высота снаряда равна s(t)=−64t2+192t 

Найдите среднюю скорость изменения высоты от t=1 

секунд до  t=1,5 

секунд.

52

Сумма денег на счете после  t 

лет, непрерывно начисляемая по ставке 4,25%, определяется по формуле  A=A0e0,0425t,

, где  A0 

— первоначальная сумма инвестиций. Найти среднюю скорость изменения баланса счета от t=1 

года до  t=2 

года, если первоначальная сумма инвестиций составляет 1000,00 долларов США.

Глоссарий

свойства пределов
сборник теорем для нахождения пределов функций путем выполнения математических операций над пределами


Эта работа находится под лицензией Creative Commons Attribution 4.0 International License.

Вы также можете бесплатно загрузить его с http://cnx.org/contents/[email protected]

Атрибуция:

  • По вопросам, касающимся этой лицензии, обращайтесь по адресу [email protected].
  • Если вы используете данный учебник в качестве библиографической ссылки, то цитировать его следует следующим образом: Колледж OpenStax, Precalculus. OpenStax CNX. http://cnx.org/contents/[email protected].
  • Если вы распространяете этот учебник в печатном формате, вы должны указать на каждой физической странице следующее указание авторства: «Скачать бесплатно на http://cnx.org/contents/fd53eae1-fa23-47c7-bb1b-972349835c3c@8. 1.»
  • Если вы повторно распространяете часть этого учебника, вы должны сохранять при каждом просмотре страницы в цифровом формате (включая, помимо прочего, EPUB, PDF и HTML) и на каждой физической печатной странице следующее указание авторства: «Загрузите бесплатно по адресу http://cnx.org/contents/[email protected]».

Помощь с ограничениями — Уроки Wyzant

Все расчеты основаны на том принципе, что мы всегда можем использовать приближения с возрастающей точностью до
.0151 найти точный ответ, например аппроксимировать кривую рядом прямых линий
в дифференциальном исчислении (чем короче линии и чем расстояние между точками
приближается к 0, тем они ближе к подобию кривой) или аппроксимировать
сферическое тело серией кубов в интегральном исчислении (по мере того, как размер кубов
становится меньше, а количество кубов внутри сферы приближается к бесконечности,
конечный результат становится ближе к фактической площади сферы).

С помощью современных технологий графики функций часто легко построить.
Основное внимание сосредоточено между геометрической и аналитической информацией и использованием
исчисления как для предсказания, так и для объяснения наблюдаемого локального и долгосрочного поведения
функции. На занятиях по исчислению ограничения обычно являются первой темой, которую вводят.

Чтобы понять работу дифференциального и интегрального исчисления, нам нужно
, чтобы понять концепцию предела. Пределы используются при дифференцировании при нахождении
приближение для
наклона прямой в конкретной точке, а также интегрирование при нахождении
площади под кривой. В исчислении пределы вводят компонент бесконечности .
Мы можем спросить себя, что происходит со значением функции, когда независимая переменная
становится бесконечно близкой к определенному значению?

График иллюстрирует нахождение предела зависимой переменной f(x) как
x приближается к c . Способ найти это — подставить значения, которые приближают 90 151 к c слева и значения, близкие к c справа.

Чтобы еще больше проиллюстрировать концепцию предела, рассмотрим последовательность чисел
x:

.

Эти значения все ближе и ближе к 2 (т.е. они приближаются к 2 как
своему пределу). Мы можем сказать, что независимо от того, какое значение мы рассматриваем, 2 является наименьшим 9.0151, которое больше, чем каждый вывод f(x) в последовательности. Когда мы возьмем
разностей этих чисел, они будут становиться все меньше и меньше. В исчислении разницу между членами последовательности и их пределом можно сделать бесконечно малой.

Иногда для нахождения предельного значения выражения достаточно просто подставить
число.

(1) Найдите предел, когда t приближается к 10 выражения

Мы запишем это, используя предельное обозначение как

В этом примере мы просто подставляем и пишем

Сложности нет, потому что M = 3t + 7 — непрерывная функция, но
бывают случаи, когда мы не можем просто так подставить.

(2) Найдите предел при приближении x к 0

Обратите внимание, что мы не можем просто заменить 0, потому что sin(0) 0
не определено и, следовательно, не является непрерывным. Не существует алгебраического процесса
для нахождения этого предела. Если мы подставим 0 вместо x , мы получим 0 0 ,
, что не определено. Однако существует метод, использующий дифференцирование (см. Правило Лопиталя). мы можем
найдите предел без использования дифференцирования, глядя на поведение функции
слева и справа от x = 0. Мы можем подставить значения, которые все ближе и ближе
к 0 слева и справа, чтобы сделать вывод, что

Способ проверить это — построить график и увидеть, что предел по мере приближения x к
равен 1.

Давайте посмотрим на увеличенное изображение этого изображения и посмотрим на его поведение как x
получается бесконечно большим и бесконечно маленьким.

Этот образ кажется знакомым? Если это так, то это потому, что это похоже на функцию
звуковой волны, где ось x — это время, а ось y — количество
децибел (громкость). Обратите внимание, что волна спадает в любом направлении, она
приближается к 0, но на самом деле никогда не устанавливается. Интересно подумать, что каждый звук 9Когда-либо созданная волна 0151 все еще существует и колеблется на бесконечно малом уровне!

(3) Рассмотрим предел, поскольку x приближается к бесконечности функции

Мы можем обнаружить, что если мы берем все большие и большие значения x , значение дробей
становится все меньше и меньше, пока не приблизится к 0. Мы говорим, что
предел 5 x при приближении x к бесконечности
равен 0:

(4) Найдите предел этой функции, когда x приближается к бесконечности.

Для этой функции не очень очевидно, каков предел. Мы могли бы подставлять
большее и большее значение x , пока не увидим, что происходит (попробуйте 100, затем
1000, затем 10000 и так далее). Мы также можем изменить выражение и использовать тот факт
, что предел, когда x приближается к бесконечности 1 x
, равен 0, чтобы найти предельное значение.

Мы делим все на x , чтобы получить выражение в форме, в которой мы можем вычислить его
.


Заметьте, что мы не можем подставить бесконечность в дробь, потому что это не
имеют математический смысл (бесконечность — это не число). 5 x
и 1 x Перейти до 0 x . 2 .

(5) Пределы также могут существовать в точках графика, где выход f(x)
является другим значением.

Мы можем видеть, что даже если график прерывистый, поскольку x = 2, мы знаем, что
существует предел, потому что график приближается к 2 слева и справа.

(6) Рассмотрим функцию f(x)= 1 x :

Как ведет себя эта функция при увеличении значения x ? Мы можем
увидеть, что график приближается к x ось, высота которой равна 0. Если вспомнить
в предварительном исчислении и алгебре, эта функция будет иметь асимптоту при y
= 0
. Мы можем сказать, что когда x приближается к бесконечности, f(x) приближается к
0.

Точно так же мы можем сказать, что когда x приближается к отрицательной бесконечности, оно также приближается к
0.

Мы можем заключить, что один над бесконечностью и один над отрицательной бесконечностью равны
0.

На самом деле, любое число, превышающее положительную или отрицательную бесконечность, будет сходиться к 0 — если
и числитель, и знаменатель не равны положительной или отрицательной бесконечности, тогда они
будут сходиться к 1.

Имейте в виду, что положительная и отрицательная бесконечность — это просто идеи . Это
почему в математических обозначениях мы используем ограничения, чтобы доказать, что число становится бесконечно большим
или малым, оно сходится к числу или не сходится вообще!

А как насчет того, когда x приближается к 0? Мы можем видеть, что по мере приближения к оси y
(x=0) справа он становится очень большим, а по мере приближения к оси y
слева он становится очень маленьким. Мы можем сделать вывод, что

Поэтому

Это невозможно! Так как предел различен слева и справа,
не существует. Вот почему деление на 0 не определено — оно равно как положительному
, так и отрицательной бесконечности!

(7) Вот геометрический пример предела. Давайте посмотрим на многоугольник, вписанный
в окружность. Если мы увеличим количество сторон многоугольника, что мы можем сказать
о многоугольнике по отношению к окружности?

По мере увеличения количества сторон многоугольник становится все ближе и ближе
до становления кругом. Если мы обобщим многоугольник как n-угольника, , где
n — количество сторон, мы можем сделать некоторые математические утверждения о
многоугольнике:

  • По мере того, как n становится больше, n-угольник становится ближе к кругу.
  • Когда n приближается к бесконечности, n-угольник приближается к окружности.
  • Предел n-угольника , когда n уходит в бесконечность, является окружностью.

Мы также можем использовать дифференцирование для решения более сложных пределов, таких как неопределенных
пределов
. Это пределы, при которых и числитель, и знаменатель приближаются к
0 или положительной или отрицательной бесконечности, например

В пределе слева, когда x приближается к 3, частное приближается к
0/0. Непонятно что делает лимит в районе х =3. В лимите на
вправо, так как x приближается к бесконечности, частное станет .
Опять же, не совсем ясно, каким будет предел при стремлении x к бесконечности. Поэтому
эти два предела считаются неопределенными.

Для решения неопределенных пределов см.
Правило Лопиталя.

Напомним, ограничения связаны с тем, что функция делает вокруг заданной точки.
Мы заметили, что пределы могут существовать, даже если функция в этой точке не существует.
Можно найти пределы, подставив предельное значение в функцию (пример 1),
используя таблицы значений (пример 2), и используя знание других пределов, чтобы найти
предел функции в заданной точке (пример 4) .

  • важно развить интуитивное понимание процесса ограничения функции
  • уметь вычислять предел с помощью алгебры
  • оценить предел по графикам или таблицам

Интуитивное введение в пределы — BetterExplained

Пределы, основы исчисления, кажутся такими искусственными и хитрыми: «Пусть x приблизится к 0, но не доберется туда, но мы будем действовать так, как будто оно там…» Тьфу.

Вот как я научился ими пользоваться:

  • Что такое лимит? Наш лучший прогноз точки, которую мы не наблюдали.
  • Как мы делаем прогноз? Приблизьтесь к соседним точкам. Если наш прогноз всегда находится между соседними точками, как бы мы ни увеличивали масштаб, это и есть наша оценка.
  • Зачем нужны ограничения? Математика имеет сценарии «черной дыры» (деление на ноль, переход к бесконечности), и ограничения дают нам оценку, когда мы не можем вычислить результат напрямую.
  • Откуда мы знаем, что мы правы? Нет. Наше предсказание, предел, не обязательно должно соответствовать действительности. Но для большинства природных явлений так оно и есть.

Ограничения позволяют задать вопрос «А что, если?». Если мы можем непосредственно наблюдать функцию при некотором значении (например, x=0 или x бесконечно растет), нам не нужен прогноз. Предел удивляется: «Если вы можете видеть все кроме единственное значение, как вы думаете, что там?».

Когда наш прогноз непротиворечив и улучшается по мере того, как мы присматриваемся , мы чувствуем в нем уверенность. И если функция ведет себя плавно, как и большинство реальных функций, предел находится там, где должна быть недостающая точка.

Ключевая аналогия: предсказание футбольного мяча

Представьте, что вы смотрите футбольный матч. К сожалению, связь прерывается:

Подтвердить! Мы пропустили то, что произошло в 4:00. Тем не менее, каков ваш прогноз относительно положения мяча?

Легко. Просто возьмите соседние моменты (3:59 и 4:01) и предскажите, что мяч окажется где-то посередине.

И… работает! Объекты реального мира не телепортируются; они перемещаются через промежуточные положения на своем пути из A в B. Наш прогноз: «В 4:00 мяч находился между своей позицией в 3:59 и 4:01». Неплохо.

С помощью замедленной камеры мы могли бы даже сказать: «В 4:00 мяч находился между своими позициями в 3:59,999 и 4:00,001».

Наш прогноз кажется верным. Можем ли мы сформулировать, почему?

  • Прогнозы совпадают при увеличении уровня масштабирования . Представьте, что диапазон 3:59–4:01 составлял 9,9–10,1 метра, но после увеличения до 3:59,999–4:00,001 диапазон расширился до 9–12 метров. О, о! Масштабирование должно сузить нашу оценку, а не сделать ее хуже! Не каждый уровень масштабирования должен быть точным (представьте, что вы смотрите игру каждые 5 минут), но чтобы чувствовать себя уверенно, должен быть некоторый порог, при котором последующие масштабирования только усиливают нашу оценку диапазона.

  • Согласие до и после. Представьте, что в 3:59 мяч находился на высоте 10 метров, катясь вправо, а в 4:01 он был на высоте 50 метров, катясь влево. Что случилось? У нас был внезапный прыжок (смена камеры?), и теперь мы не можем определить положение мяча. У кого был мяч в 4:00? Эта двусмысленность разрушает нашу способность делать уверенные прогнозы.

С учетом этих требований мы могли бы сказать: «В 4:00 мяч находился на расстоянии 10 метров. Эта оценка подтверждается нашим первоначальным зумом (3:59-4:01, что составляет от 9,9 до 10,1 метра) и следующее (3:59,999-4:00,001, что составляет от 9,999 до 10,001 метра)».

Ограничения — это стратегия уверенных прогнозов.

Изучение интуиции

Давайте пока не будем приводить математические определения. Для каких вещей в реальном мире мы хотим получить точный прогноз, но не можем легко измерить?

Какова длина окружности?

Найти число пи «экспериментально» сложно: взять нить и линейку?

Мы не можем измерить фигуру с, казалось бы, бесконечными сторонами, но мы можем задаться вопросом: «Существует ли предсказанное значение числа пи, которое всегда будет точным, если мы продолжаем увеличивать стороны?»

Архимед вычислил, что число пи имеет диапазон

, используя следующий процесс:

Это было предшественником исчисления: он определил, что число пи было числом, которое оставалось в его постоянно сужающихся границах. В настоящее время у нас есть современные предельные определения числа пи.

Как выглядит совершенно непрерывный рост?

e, одно из моих любимых чисел, можно определить следующим образом:

Мы не можем легко измерить результат бесконечно сложного роста. Но если бы мы могли сделать прогноз , существует ли единственная скорость, которая всегда точна? Кажется, это около 2,71828…

Можем ли мы использовать простые формы для измерения сложных?

Круги и кривые измерить сложно, а прямоугольники легко. если мы может ли использовать бесконечное количество прямоугольников для имитации криволинейной области, можем ли мы получить результат, выдерживающий бесконечные проверки? (Может быть, мы сможем найти площадь круга.)

Можем ли мы найти скорость в данный момент?

Скорость забавная: нужно измерение до и после (пройденное расстояние/затраченное время), но разве мы не можем иметь скорость в отдельные моменты времени? Хрм.

Ограничения помогают ответить на эту загадку: предсказать свою скорость при путешествии в соседнее мгновение. Затем задайте «невозможный вопрос»: какова ваша прогнозируемая скорость, когда расстояние до соседнего мгновения равно нулю?

Примечание: лимит не панацея от всех бед. Мы не можем предположить, что он существует, и не может быть ответа на каждый вопрос. Например: число целых чисел четное или нечетное? Количество бесконечно, и ни «четное», ни «нечетное» предсказание не остаются точными, если мы считаем выше. Не существует хорошо поддерживаемого прогноза.

Для числа пи, е и основ исчисления умные умы сделали доказательства, чтобы определить, что «да, наши предсказанные значения становятся более точными, чем ближе мы смотрим». Теперь я вижу почему пределы так важны: они подтверждают наши прогнозы.

Математика: формальное определение предела

Пределы — это хорошо обоснованные предсказания. Вот официальное определение:

. означает, что для всех действительных ε > 0 существует действительное δ > 0 такое, что для всех x с 0

Давайте сделаем это читаемым:

Математический английский Человеческий английский
Когда мы «сильно предсказываем», что f(c) = L, мы имеем в виду
для всех реальных значений ε > 0 для любого желаемого предела погрешности (+/- . 1 метра)
существует реальное значение δ > 0 секунды)
таким образом, что для всех x с 0, где прогноз остается точным в пределах погрешности

Здесь есть несколько тонкостей:

  • Уровень масштабирования (дельта, δ) является входом функции, т.е. время в видео
  • Погрешность (эпсилон, ε) — это максимальное значение, которое выход функции (положение шарика) может отличаться от нашего прогноза на всем уровне масштабирования
  • Условие абсолютного значения (0 < |x − c| < δ) означает, что должны работать положительные и отрицательные смещения, и мы пропускаем саму черную дыру (когда |x – c| = 0).

Мы не можем оценить ввод черной дыры, но мы можем сказать: «За исключением отсутствующей точки, весь уровень масштабирования подтверждает предсказание $f(c) = L$». А поскольку $f(c) = L$ верно для любая погрешность , которую мы можем найти, мы чувствуем себя уверенно.

Можем ли мы иметь несколько прогнозов? Представьте, что мы предсказали L1 и L2 для f(c). Между ними есть некоторая разница (назовем ее 0,1), поэтому есть некоторая погрешность (0,01), которая выявляет более точную. Выход каждой функции в диапазоне не может быть в пределах 0,01 от обоих прогнозов. Либо у нас есть единственное бесконечно точное предсказание, либо его нет.

Да, мы можем быть милыми и попросить «ограничение левой руки» (прогноз до события) и «ограничение правой руки» (прогноз после события), но у нас есть реальный предел только тогда, когда они согласны.

Функция является непрерывной, если она всегда соответствует прогнозируемому значению (и прерывистой, если нет):

Исчисление обычно изучает непрерывные функции, играя в игру «Мы делаем прогнозы, но только потому, что знаем, что они будут правильными».

Математика: демонстрация существования предела

У нас есть требования для надежного прогноза. Вопросы, в которых вас просят «доказать, что предел существует», просят вас обосновать свою оценку.

Например: Докажите, что предел при x=2 существует для

Первая проверка: нужен ли вообще лимит? К сожалению, мы это делаем: просто подставив «x = 2», мы получим деление на ноль. Дратс.

Но интуитивно мы видим, что один и тот же «ноль» (x – 2) может быть отменен сверху и снизу. Вот как танцевать это опасное танго:

  • Предположим, что x находится в любом месте , кроме 2 (Должно быть! Мы делаем прогноз извне.)
  • Затем мы можем отменить (x – 2) сверху и снизу, так как это не ноль.
  • Осталось f(x) = 2x + 1. Эту функцию можно использовать вне черной дыры.
  • Что предсказывает эта более простая функция? Что f(2) = 2*2 + 1 = 5,

Итак, f(2) = 5 — это наш прогноз. Но вы видели подлость? Мы притворились, что x не равно 2 [чтобы разделить (x-2)], а затем подставили 2 после того, как этот неприятный элемент исчез! Подумайте об этом так: мы использовали простое поведение вне события , чтобы предсказать грубое поведение при событии .

Мы можем доказать, что эти махинации дают надежное предсказание, и что f(2) = 5 бесконечно точен.

Для любого порога точности (ε) нам нужно найти «диапазон масштабирования» (δ), в котором мы остаемся в пределах заданной точности. Например, можем ли мы оставить оценку в пределах +/- 1,0?

Конечно. Нам нужно выяснить, где

так

Другими словами, x должен оставаться в пределах 0,5 от 2, чтобы поддерживать исходное требование точности 1,0. Действительно, когда x находится между 1,5 и 2,5, f(x) изменяется от f(1,5) = 4 до f(2,5) = 6, оставаясь +/- 1,0 от нашего предсказанного значения 5,9.0003

Мы можем обобщить любой допуск на ошибку (ε), подставив его вместо 1.0 выше. Получаем:

Если наш уровень масштабирования «δ = 0,5 * ε», мы останемся в пределах исходной ошибки. Если наша ошибка равна 1,0, нам нужно увеличить масштаб до 0,5; если это 0,1, нам нужно увеличить масштаб до 0,05.

Эта простая функция была удобным примером. Идея состоит в том, чтобы начать с начального ограничения (|f(x) – L| < ε), подставить f(x) и L и найти расстояние от точки черной дыры (|x – c| < ?). Часто это упражнение по алгебре.

Иногда вас просят просто найти предел (подставьте 2 и получите f(2) = 5), в других случаях вас просят доказать, что предел существует, т. е. провернуть эпсилон-дельта-алгебру.

Переворачивание нуля и бесконечности

Бесконечность при использовании в пределе означает «растет без остановки». Символ ∞ является числом не больше, чем предложение «растет без остановки» или «мои запасы трусов истощаются». Это понятия, а не числа (для нашего уровня математики, Алеф только меня).

При использовании ∞ в пределе мы спрашиваем: «Поскольку x растет без остановки, можем ли мы сделать прогноз, который останется точным?». Если есть предел, это означает, что прогнозируемое значение всегда подтверждается, как бы далеко мы ни смотрели.

Но бесконечность мне все равно не нравится, потому что я ее не вижу. Но я вижу ноль. С лимитами можно переписать

как

Вы можете пойти на хитрость и определить y = 1/x, заменить элементы в формуле, а затем использовать

, так что это снова похоже на обычную проблему! (Примечание от Тима в комментариях: предел идет справа, так как x стремится к положительной бесконечности). Я предпочитаю такое расположение, потому что я могу видеть место, где мы сужаемся (у нас всегда заканчивается бумага, когда мы рисуем бесконечную версию).

Почему ограничения не используются чаще?

Представьте себе ребенка, который понял, что «нуль в конце» увеличивает число в 10 раз. Есть 5? Запишите «5», затем «0» или 50. Есть 100? Сделать 1000. И так далее. 92$» без строгого обоснования. Тем не менее, судя по его неофициальным результатам, крутятся двигатели и летают самолеты.

Педагогическая ошибка в исчислении заключается в создании препятствия вроде «Вы должны знать Пределы™, прежде чем ценить исчисление», когда ясно, что изобретатели исчисления этого не знали. Я бы предпочел эту прогрессию:

  • Исчисление задает, казалось бы, невозможные вопросы: когда прямоугольники могут измерять кривую? Можем ли мы обнаружить мгновенное изменение?
  • Ограничения дают стратегию ответов на «невозможные» вопросы («Если вы можете сделать прогноз, который выдерживает бесконечную проверку, мы скажем, что все в порядке»). 2$), точно так же, как мы запоминаем сокращения для правил, которые мы проверили с помощью умножения (добавление нуля означает умножение на 10). Но все же приятно знать, почему ярлыки оправданы.

Ограничения — не единственный инструмент для проверки ответов на невозможные вопросы; бесконечно малые тоже работают. Ключ в том, чтобы понять , что мы пытаемся предсказать, , а затем изучить правила предсказания.

Счастливая математика.

Другие сообщения из этой серии

  1. Нежное введение в изучение исчисления
  2. Понимание исчисления с помощью метафоры банковского счета
  3. Доисторическое исчисление: открытие Пи
  4. Аналогия с исчислением: интегралы как умножение
  5. Исчисление: построение интуиции для производной
  6. Как понимать деривативы: произведение, мощность и правила цепочки
  7. Как понимать производные: правило частных, показатели степени и логарифмы
  8. Интуитивное введение в ограничения
  9. Интуиция для ряда Тейлора (аналогия ДНК)
  10. Зачем нужны пределы и бесконечно малые числа?
  11. Обучение исчислению: преодоление нашей искусственной потребности в точности
  12. Дружеский разговор о том, 0,999.

Добавить комментарий

Ваш адрес email не будет опубликован. Обязательные поля помечены *